Physics 1 Our Project
Physics 1 Our Project
Physics 1 Our Project
MORONG RIZAL
PROJECT MIDTEM
BT-ELT 3-4
(PHYSICS 1)
Solution: Using the equation of motion, v = u + at, where v is the final velocity, u is the
initial velocity, a is the acceleration, and t is the time:
v = 25 m/s + (2 m/s²) (10 s)
v = 25 m/s + 20 m/s
v = 45 m/s
2. Problem: An object is dropped from a height of 50 meters. Calculate the time it takes
for the object to hit the ground.
Solution: Using the formula h = (1/2)gt², where h is the height, g is the acceleration due
to gravity (approximately 9.8 m/s²), and t is the time:
50 m = (1/2) (9.8 m/s²) t²
t² = (2*50 m) / (9.8 m/s²)
t² ≈ 10.204
t ≈ √10.204
t ≈ 3.2 s
Solution: Using Hooke's Law, F = kx, where F is the force, k is the spring constant, and
x is the displacement:
20 N = (100 N/m) x
x = 20 N / 100 N/m
x = 0.2 m
4. Problem: A ball is thrown vertically upwards from the ground with an initial velocity of
15 m/s. Calculate the maximum height reached by the ball.
Solution: The maximum height is reached when the ball's velocity becomes zero.
Using the equation v² = u² + 2as, where v is the final velocity, u is the initial velocity, a
is acceleration, and s is displacement:
0 = (15 m/s)² + 2(-9.8 m/s²)s
0 = 225 m²/s² - 19.6 m/s² s
19.6 m/s² s = 225 m²/s²
s ≈ 11.5 m
Solution: Using the equation of motion, v = u + at, where v is the final velocity, u is the
initial velocity, a is the acceleration, and t is the time:
30 m/s = 0 m/s + a(10 s)
30 m/s = 10 a
a = 30 m/s / 10 s
a = 3 m/s²
7. Problem: An object with an initial velocity of 5 m/s comes to rest in 10 seconds due
to an acceleration of -0.5 m/s². Calculate the displacement of the object during this
time.
Solution: Using the equation of motion, v = u + at, where v is the final velocity, u is the
initial velocity, a is the acceleration, and t is the time:
0 m/s = 5 m/s + (-0.5 m/s²)(10 s) + s
s = -5 m/s² * 10 s
s = -50 m
Solution: The frequency, f, is the inverse of the period: f = 1 / T, where T is the period.
f=1/2s
f = 0.5 Hz
Solution: The work done, W, is given by the formula W = Fd, where F is the force
applied, and d is the displacement.
W = 100 N * 20 m
W = 2000 J (Joules)
ANTONIL, DON HENLEY
Solution: According to Newton's first law, F=ma, where F is the force, m is the mass,
and a is the acceleration. Substituting the given values, we have F=5kg∗2m/s 2 =10N.
Solution: Again, using Newton's second law, F=ma, you can rearrange the formula to
find acceleration: a= m/F . Substituting the values, a= 10kg 30N =3m/s 2.
Problem 3: Newton's Third Law Problem: Two cars collide. Car A exerts a force of
6000 N to the east on car B. What force does car B exert on car A?
Solution: According to Newton's third law, action and reaction forces are equal in
magnitude but opposite in direction. So, car B exerts a force of 6000 N to the west on
car A.
Solution: The force of kinetic friction is given by Fk =μk ∗N, where N is the normal
force. The normal force is equal to the weight of the object, N=mg, where m is the
mass and g is the acceleration due to gravity. So, N=50kg∗9.8m/s 2 =490N, and F k
=0.3∗490N=147N.
Solution: The tension in the rope is equal to the weight of the hanging mass, which is
T=mg=4kg∗9.8m/s 2 =39.2N.
Solution: The acceleration can be found using a=g∗sin(θ), where g is the acceleration
due to gravity and θ is the angle of the incline. Substituting the values, a=9.8m/s 2
∗sin 30°)=4.9m/s 2 .
Solution: The work done is given by W=Fd, where F is the force and d is the
displacement. Substituting the values, W=100N∗5m=500J(joules).
Problem 10: Conservation of Momentum Problem: Two ice skaters of equal mass,
initially at rest, push off each other on an ice rink. If one skater has a velocity of 3 m/s
to the right, what is the velocity of the other skater?
Solution: According to the conservation of momentum, the total momentum before the
push is zero since they are initially at rest, and it remains zero after the push.
Therefore, the other skater must have a velocity of -3 m/s to the left to keep the total
momentum zero.
ANTONIO, ADRIAN
Numerical Problems
1. Calculate the work done by a force of 30 N in lifting a load of 2kg to a height of 10m
(g = 10ms-2)
Answer:
Given :
Force mg = 30 N ; height = 10 m
Work done to lift a load W = ?
W = F.S (or) mgh
= 30x10
W = 300 J
Ans: 300J
2. A bob of mass m is attached to one end of the rod of negligible mass and length r,
the other end of which is pivoted freely at a fixed center O as shown in the figure. What
initial speed must be given to the object to reach the top of the circle? (Hint: Use law of
conservation of energy). Is this speed less or greater than speed obtained in the
section 4.2.9?
v12 = vL2 - 2gr (1 - cos θ)
v12 = 5gr - 2gr (l-cosθ)
If θ = 60°
v12 = 5gr-2gr(l – 1/2)
v12 = 5gr - gr => v12 = 4gr
v1 = √[4gr] ms-
Ans: √4gr ms-1
3. Two different unknown masses A and B collide. A is initially at rest when B has a
speed v. After collision B has a speed v/2 and moves at right angles to its original
direction of motion. Find the direction in which A moves after collision.
Answer:
Momentium is censerved in both × and y direction.
In x - direction
MBVB = O + MAVA' cosɸ ...(l)
In y - direction
O = MBVB' - MAVA'sinɸ ...(2)
(2)/(1) tanɸ = VB’/VB = 1/2
tan ɸ = ½
ɸ = 26.6° (or) 26° 36' [1° = 60']
Ans: θ = 26° 33′
6.If a 15kg object is subjected to a force of 175N to the east and another force of
1020N to the west, what is the object’s acceleration?
Possible Answers:
113m/s2
1.4m/s2
5.6m/s2
56m/s2
Correct answer:
56m/s2
7.Write what you are given, your assumptions, and what the problem is asking you to
find
Keep in mind that the problem may give you more information than you need to solve it
m = 5 kg v₁ = 0 m/s
Y₁ = 8 m 9 = 9.8 m/s²
Yt = 0 m V₁ = ?
9. A car of mass is initially at rest, and then accelerates at for . What is the kinetic
energy of the car at time ?
Possible Answers:
Correct answer:
Explanation:
The first step will be to find the final velocity of the car. We know the acceleration and
time, so we can find the final velocity using kinematics. The initial velocity is zero, since
the car starts at rest.
10. A spring is compressed from equilibrium by a force of . What is the spring constant
for this spring?
Possible Answers:
Correct answer:
Explanation:
We use Hooke's law equation to relate the force, displacement, and spring constant:
RAYMART B. BADILLOS
Solution:
Here is an illustration of the setup.
The variables we know are:
x0 = 5 m
v0 = 3 m/s
a = 2 m/s2
(Part a) Where is the block at t = 2 seconds?
Equation 1 is the useful equation for this part.
x = x0 + v0t + ½at2
Substitute t = 2 seconds for t and the appropriate values of x0 and v0.
x = 5 m + (3 m/s)(2 s) + ½(2 m/s2)(2 s)2
x=5m+6m+4m
x = 15 m
The block is at the 15 meter mark at t = 2 seconds.
There are two ways you could do this. You could use Equation 1 and solve for t using
the value you calculated in part c of the problem, or you could use equation 2 and
solve for t. Equation 2 is easier.
v = v0 + at
10 m/s = 3 m/s + (2 m/s2)t
7 m/s = (2 m/s2)t
7⁄2 s = t
It takes 7⁄2 s or 3.5 s to get to the 27.75 m mark.
One tricky part of this type of problem is you have to pay attention to what the question
is asking for. In this case, you were not asked how far the block travelled, but where it
is. The reference point is 5 meters from the origin point. If you needed to know how far
the block travelled, you would have to subtract the 5 meters.
2. A motorist is speeding along at 120 km/hr when he sees a squirrel on the road 200
meters in front of him. He tries to stop, but it takes 12 seconds for his car to stop.
(a) What is the acceleration of the car? (assume acceleration was constant)
(b) Does the squirrel survive?
(c) How fast was the car moving at 100 meters?
Solution:
This shows the conditions of the vehicle at the beginning (t = 0 s) and when the car
has come to a stop (v = 0 km/hr)
v0 = 33.33 m/s
The car is stopped at the end, so the final velocity is equal to zero.
0 = 33.33 m/s + a(12 s
-33.33 m/s = a(12 s)
a = -2.78 m/s2
Note the acceleration is negative. This means it is slowing down the vehicle as motion
progresses in the positive direction. Just what you would expect in a problem where
the vehicle is slowing down.
3. A speeding motorist travelling at 120 km/hr passes a stopped police car. The police
car immediately begins to chase the speeder, accelerating at a constant 2.5 m/s2.
(a) How long does it take for the police car to intercept the speeder?
(b) How far did the police car travel before catching up to the speeder?
(c) How fast was the police car travelling when it intercepts the speeder?
Solution:
This illustration shows the conditions of the vehicles at the beginning of the problem
and the time when the police car intercepts the speeder.
(Part a) How long does it take for the police car to intercept the speeder?
First, let’s look at the police car’s equations of motion.
xPI = x0P + V0Pt + ½at2
since the police car starts at 0 and at rest, v0P=0 then
xP = ½at2
vPI = v0P + at
vPI = at
Now for the speeder’s car’s equations of motion.
xS = x0S + V0St + ½at2
x0 = 0 and the speeder is not accelerating, a = 0, therefore
xS = v0St
vS = v0S + at
vS = V0S
vS = 120 km/hr
convert to m/s since our acceleration is in m/s2 and it probably won’t take hours for the
police car to catch up.
plug in the values for a = 2.5 m/s2 and v0S = 33.3 m/s and get
t= 26.6 s
It takes 26.6 seconds for the police car to catch up and intercept the speeder.
Part b) How far did the police car travel before catching up to the speeder?
Now that we know the time, we can find the distance. From the police car’s position
equation above:
xPI = ½at2
xPI = ½(2.5 m/s2)(26.6)2
xPI = 888.4 m
The police car travelled 888.4 m before it intercepted the speeder.
(Part c) How fast was the police car travelling when it intercepts the speeder?
Again, using the time and the police car’s velocity equation from above:
vPI = at
vPI = (2.5 m/s2)(26.6 s)
vPI = 66.7 m/s
The police car was travelling at 66.7 m/s when it intercepted the speeder. If you
convert it to km/hr, the speed of the police car is 239.9 km/hr. Talk about speeding
vehicles.
4. The human eye’s optic nerve needs 2 x 10-17 joules of energy to trigger a series of
impulses to signal the brain there is something to see. How many photons of 475 nm
blue light is needed to trigger this response?
Solution: We are given the amount of energy needed to trigger the optic nerve and the
wavelength of light.
First, let’s figure out how much energy is in a single photon of the blue light. We are
given the wavelength as 475 nm. Before we go any further, let’s convert this to meters.
1 nm = 10-9 m
Using this relationship, convert 475 nm to meters
x m = 4.75 x 10-7 m
Now we can use the energy formula from above
E = hc / λ
Plug in the variables
E = (6.626 x 10-34 m2kg/s)(3 x 108 m/s) / 4.75 x 10-7 m
Solve for E
E = 4.18 x 10-19 J
This is the energy of a single photon of 475 nm blue light. We need 2 x 10-17 J of
energy to begin the process.
x photons = 2 x 10-17 J
1 photon = 4.18 x 10-19 J
Divide one equation into the other to get
Solve for x
x = 47.8 photons
Since you can’t have partial photons, we need to round this answer up to the nearest
whole photon count. 47 photons isn’t enough, so one more is needed to get over the
threshold energy.
x = 48 photons
Answer: It takes 48 photons of 475 nm blue light to trigger the optic nerve.
5. Grandfather clocks are decorative clocks with a pendulum measuring out the
passing of a second. How long of a pendulum is needed to have a period of 1 second?
Use 9.8 m/s2 for the acceleration due to gravity.
Square both sides to get rid of the radical
L = 0.25 m
Answer: A simple pendulum with a period of 1 second will have a length of 0.25
meters or 25 centimeters.
It is a good idea to write all your units along with your values with these types of
problems. This can catch simple math errors when you expect a length for your answer
and you happen to have length squared or 1/length. It can save you time in the long
run.
6. If it takes 41000 joules of heat to melt 200 grams solid copper to liquid copper, what
is the heat of fusion of copper?
7. Question: Liquid sulfur vaporizes at 445 °C. If it takes 28125 J to convert 20 grams
of 445 °C liquid sulfur to 445 °C gaseous sulfur, what is the heat of vaporization of
sulfur?
Solution: Plug what we know into the heat formula from above.
m = 50 grams
ΔHv = 2257 J/gram
Q = m · ΔHv
Q = (50 g) · (2257 J/g)
Q = 112850 J = 112.85 kJ
In calories:
ΔHv = 540 cal/g
Q = m · ΔHv
Q = (50 g) · (540 cal/g)
Q = 27000 cal = 27 kcal
Answer: It takes 112850 Joules or 27000 calories of heat to convert 50 grams of liquid
water into gaseous water or steam.
9. Question: A 500 gram cube of lead is heated from 25 °C to 75 °C. How much
energy was required to heat the lead? The specific heat of lead is 0.129 J/g°C.
Plug these values into the specific heat equation from above.
Q = mcΔT
Q = (500 grams)·(0.129 J/g°C)·(50 °C)
Q = 3225 J
Answer: It took 3225 Joules of energy to heat the lead cube from 25 °C to 75 °C.
10. Question: A 25-gram metal ball is heated 200 °C with 2330 Joules of energy.
What is the specific heat of the metal?
Question 1
Answer: A
First, we’ll need to rely on Newton’s Third Law of Motion. This law states that two
bodies exert forces of equal magnitude and
opposite direction on each other. In this
case, the force that block Y exerts on block
X has magnitude �����, and the force
that block X exerts on block Y has
magnitude ���. By Newton’s Third Law,
these forces should be equal in magnitude.
So, we have that �����=���.
Because block Y was initially moving downwards, then slowed down to a stop, we
know that the acceleration is in the opposite direction of motion. So, the acceleration is
in the upwards direction. Then, because ��� is in the same direction as acceleration
and ��� is in the opposite direction of acceleration, we have
that ����=���−���.
Then, ���=����+���.
Answer: B
We can first analyze the potential energy of the block-Earth system at points A and C.
We have the following formula:
So, ��=��. This makes sense because the block is at the same height at points A
and C, so the potential energy of the block-Earth system (which is dependent on
height) is the same.
Next, let’s look at the kinetic energy by analyzing the total energy. We can assume that
the total energy of the system is conserved since the track is frictionless.
So, the total energy at point A is equal to the total energy at point C. Next, let’s
consider what types of energy we have at each point. We know that we have to
consider gravitational potential energy and kinetic energy at each point. But, we also
have to look at the potential energy given by the spring at point A.
Then, ��+��=�� (because ��=��). So, we can clearly see that ��<��,
and the correct answer choice is B.
Question 3
Answer: C
To answer this question, we can once again analyze the energy at both points. Since
friction is negligible, we know that the total energy at both points is conserved, and
therefore equal.
So, the total energy at the beginning of the track consists only of gravitational potential
energy. The total energy at the end of the track consists of both gravitational potential
energy and kinetic energy.
In the second equation, � is the kinetic energy, � is the mass, and � is the speed.
�=[�(0.4)+12�2]�.
Question 4
Answer: B
In this equation, Δ� is the momentum, � is the force, and Δ� is the change in time.
So, if we compute the area under the curve for each object, we’ll be able to determine
the change in momentum for each. Since the areas under both curves have the shape
of a triangle, we can simply use the triangle area formula to compute momentum.
Object X: ��=12(�0)(�0)
Object Y: ��=12(2�0)(�02)=12(�0)(�0)
Answer: D
Note that the force which contributes to work (�) is the force (�) that acts parallel to
the direction (�) of motion.
In this case, since the force is exerted upwards, we have that the work done by the
person is positive if the motion is upwards and negative if the motion is downwards.
So, the correct answer choice is D. Notice that answer choice C is incorrect since even
if the box is moving at a constant speed, the person does negative work on the box
because the applied force is opposite the direction of motion.
Question 6
Answer: A
This problem is tricky in that we’ll need to work with equations involving unknowns
since we’re not really given many concrete numbers. So, our strategy will be to analyze
the equations in terms of the unknowns in order to compare them.
Let’s first look at the relevant equations for gravitational force and circular motion:
We can first solve for the gravitational force on each satellite, using the first equation:
�1=��0��(2��)2=14(��0��(��)2)
�2=�2�0��(3��)2=29(��0��(��)2)
Next, we’ll need to use the second equation to compute the speed of each satellite. In
this case, the centripetal force is caused by the gravitational force, so we have
that ��=�� for each satellite.
Then, because ��=�0��, we can solve for the centripetal acceleration for each
satellite using the equation ��=���0=���0.
��1=14(���(��)2)
��2=29(���(��)2)
��1=14(���(��)2)=(�1)22��
��2=29(���(��)2)=(�1)23��
(�1)2=12(�����)
(�2)2=13(�����)
Question 7
Answer: B
This question wants us to analyze the momentum of the two carts. Because
momentum is conserved, we can analyze the momentum before and after the collision
and set it equal to each other.
Before the collision, because only the left is moving, we have that: ��=���
After the collision, since both carts are moving together at the same speed, we have
that: ��=(�+�)��.
Next, the student wants to create a graph of �� as a function of ��. So, let’s solve
for �� to determine the slope of the line:
��=(��+�)��
Question 8
Answer: A
So, the total angular momentum before and after the collision should be the same.
This means that the correct answer choice is either A or B.
Also, let’s use this fact to set up equations in order to learn more about the system.
The equations that we’ll use are:
Because momentum is conserved, the momentums before and after are equivalent.
Before:
Momentum: ��=���
Angular momentum: ��=���
After:
Momentum: ��=2���
Angular momentum: ��=2���
Because disks A and B are identical, they have the same mass and moment of inertia,
so after the collision, we have double the mass and double the moment of inertia.
Using ��=�� and ��=�� , we can solve for �� and �� to get that:
��=12��
��=12��
Now, we can use this information to analyze the total kinetic energy of the system.
Let’s use the following equations:
The first equation is for translational kinetic energy and the second equation is for
rotational kinetic energy, and we’ll need to consider both to compute the total kinetic
energy.
��=12��� 2+12��� 2
Then, the total kinetic energy after the collision is:
��=12(2�)�� 2+12(2�)�� 2
=��� 2+��� 2
=�(12��)2+�(12��)2
=12(12��� 2+12��� 2)
=12��
Question 9
Answer: B, D
For answer choice A, we’ll need to note that the satellite’s rotational kinetic energy is
dependent on the radius. But, since the orbit is elliptical, the radius is changing. Then,
the kinetic energy is not constant, so this statement is false.
For answer choice B, we need to recall that in elliptical orbit motion, angular
momentum is always constant, so this statement is true.
For answer choice C, recall that linear momentum is dependent on mass and velocity.
Though the mass is constant, the velocity is constantly changing since the satellite is
moving in an ellipse (which means the radius is changing and thus so is velocity). This
means that the linear momentum is not constant and this statement is false.
So, this quantity is the largest when � is the largest, which means the statement is
true.
Question 10
Solution:
Student 1 claims that the sphere can’t move in a circle because none of the forces
acting on it are in the direction of point C. But, what the student is forgetting is that the
centripetal force is a net force, which means it’s the accumulation of all forces acting
on the object. So, even though none of the individual forces are pointing in the
direction of point C, their sum could be pointing in that direction. Also, the tension force
exerted by the string on the sphere has a horizontal component in the direction of C
which contributes to the centripetal force.
Student 2 claims that the tension force exerted by the string is less than the weight of
the sphere because of the angle, which means it should be moving downwards. But,
what the student doesn’t realize is that the magnitude of the tension forces and the
weight are not necessarily equal. So, the magnitude of the tension force could be large
enough that the vertical component of it is still larger than or equal to the weight (even
with the angle).
CALLEJA, MARK ANTHONY
1.An airplane accelerates down a runway at 3.20 m/s2 for 32.8 s until it finally lifts off
the ground. Determine the distance traveled before takeoff.
Given:
a = +3.2 m/s2
t = 32.8 s
vi = 0 m/s
Find:
d = ??
d = vi*t + 0.5*a*t2
d = (0 m/s)*(32.8 s)+ 0.5*(3.20 m/s2)*(32.8s)2
d = 1720 m
2.A car starts from rest and accelerates uniformly over a time of 5.21 seconds for a
distance of 110 m. Determine the acceleration of the car.
Given:
d = 110 m
t = 5.21 s
vi = 0 m/s
Find:
a = ??
d = vi*t + 0.5*a*t2
110 m = (0 m/s)*(5.21 s)+ 0.5*(a)*(5.21 s)2
110 m = (13.57 s2)*a
a = (110 m)/(13.57 s2)
a = 8.10 m/ s2
3. A race car accelerates uniformly from 18.5 m/s to 46.1 m/s in 2.47 seconds.
Determine the acceleration of the car and the distance traveled.
Given:
vi = 18.5 m/s
vf = 46.1 m/s
t = 2.47 s
Find:
d = ??
a = ??
a = (Delta v)/t
a = (46.1 m/s - 18.5 m/s)/(2.47 s)
a = 11.2 m/s2
d = vi*t + 0.5*a*t2
d = (18.5 m/s)*(2.47 s)+ 0.5*(11.2m/s2)*(2.47s)2
d = 45.7 m + 34.1 m
d = 79.8 m
(Note: the d can also be calculated using the equation vf2 = vi2 + 2*a*d)
4. A race car accelerates uniformly from 18.5 m/s to 46.1 m/s in 2.47 seconds.
Determine the acceleration of the car and the distance traveled.
Given:
vi = 18.5 m/s
vf = 46.1 m/s
t = 2.47 s
Find:
d = ??
a = ??
a = (Delta v)/t
a = (46.1 m/s - 18.5 m/s)/(2.47 s)
a = 11.2 m/s2
d = vi*t + 0.5*a*t2
d = (18.5 m/s)*(2.47 s)+ 0.5*(11.2 m/s2)*(2.47 s)2
d = 45.7 m + 34.1 m
d = 79.8 m
(Note: the d can also be calculated using the equation vf2 = vi2 + 2*a*d)
5. A plane has a takeoff speed of 88.3 m/s and requires 1365 m to reach that speed.
Determine the acceleration of the plane and the time required to reach this speed.
Given:
vi = 0 m/s
vf = 88.3 m/s
d = 1365 m
Find:
a = ??
t = ??
vf2 = vi2 + 2*a*d
(88.3 m/s)2 = (0 m/s)2 + 2*(a)*(1365 m)
7797 m2/s2 = (0 m2/s2) + (2730 m)*a
7797 m2/s2 = (2730 m)*a
(7797 m2/s2)/(2730 m) = a
a = 2.86 m/s2
vf = vi + a*t
88.3 m/s = 0 m/s + (2.86 m/s2)*t
(88.3 m/s)/(2.86 m/s2) = t
t = 30. 8 s
6. With what speed in miles/hr (1 m/s = 2.23 mi/hr) must an object be thrown to reach a
height of 91.5 m (equivalent to one football field)? Assume negligible air resistance.
Find:
d = ??
d = vi*t + 0.5*a*t2
d = (0 m/s)*(3.41 s)+ 0.5*(-9.8 m/s2)*(3.41 s)2
d = 0 m+ 0.5*(-9.8 m/s2)*(11.63 s2)
d = -57.0 m
8. It was once recorded that a Jaguar left skid marks that were 290 m in length.
Assuming that the Jaguar skidded to a stop with a constant acceleration of -3.90 m/s2,
determine the speed of the Jaguar before it began to skid.
Given:
a = -3.90 m/s2
vf = 0 m/s
d = 290 m
Find:
vi = ??
vf2 = vi2 + 2*a*d
(0 m/s)2 = vi2 + 2*(-3.90 m/s2)*(290 m)
0 m2/s2 = vi2 - 2262 m2/s2
2262 m2/s2 = vi2
vi = 47.6 m /s
9.A dragster accelerates to a speed of 112 m/s over a distance of 398 m. Determine
the acceleration (assume uniform) of the dragster.
Given:
vi = 0 m/s
vf = 112 m/s
d = 398 m
Find:
a = ??
vf2 = vi2 + 2*a*d
(112 m/s)2 = (0 m/s)2 + 2*(a)*(398 m)
12544 m2/s2 = 0 m2/s2 + (796 m)*a
12544 m2/s2 = (796 m)*a
(12544 m2/s2)/(796 m) = a
a = 15.8 m/s2
10.If Michael Jordan has a vertical leap of 1.29 m, then what is his takeoff speed and
his hang time (total time to move upwards to the peak and then return to the ground)?
Given:
a = -9.8 m/s2
vf = 0 m/s
d = 1.29 m
Find:
vi = ??
t = ??
vf2 = vi2 + 2*a*d
(0 m/s)2 = vi2 + 2*(-9.8 m/s2)*(1.29 m)
0 m2/s2 = vi2 - 25.28 m2/s2
25.28 m2/s2 = vi2
vi = 5.03 m/s
To find hang time, find the time to the peak and then double it.
vf = vi + a*t
0 m/s = 5.03 m/s + (-9.8 m/s2)*tup
-5.03 m/s = (-9.8 m/s2)*tup
(-5.03 m/s)/(-9.8 m/s2) = tup
tup = 0.513 s
hang time = 1.03 s
CAPATI, CHRISTOPHER
The maximum range is obtained by setting constant. This gives cos:20-)=0 ord/dphi
(R) = 0holding , o and g
4. As the outer walls are equal in height () they are equally distant (c) from the
extremities of the parabolic trajectory whose general form may be written as(Fig. 1.20)
Fig. 1.2029
y = ax - b * x ^ 2
yDatx-R-nr, when R is the range
The trajectory passes through the top of the three walls whose coordinates are(ch). ( c
+ r ,/ h). (c+3r. h), respectively. Using these coordinates in (1).
we get three equations
h = ac - b * c ^ 2
15b = a(c + r) - b * (c + r) ^ 2
7k = a(c + 3r) - b * (c + 3r) ^ 2
Combining (2), (3), (4), (5) and (6) and solving we get n = 4,
5. The equation to the parabolic path can be written as1 Kinematics and Statics
y = ax - b * x ^ 2with a = tan thetab = 8/(2mu ^ 2 * cos^2 theta)
Taking the point of projection as the origin, the coordinates of the two openings in the
windows are (5, 5) and (11, 7), respectively. Using these coordinates in (1) we get the
equations
5 = 5a - 25b
7 = 11a - 121b
with the solutions, a = 1.303 and 0.0606. Using these values in (2), wefind theta = 52.5
deg and x = 14.8m / s
8. the quantity (H) represents the height through which the monkey drops from the tree
and the right-hand side of the last equation gives the time for a free fall. Therefore, the
bullet would hit the monkey independent of the bullet'sinitial velocity.
9. (i) T-Zu sino 2 x 800 sin 60-141.4s(iR = (a ^ 2 * sin 2a)/mu = ((800) ^ 7 * sin(2 *
60))/9.8 =5.6568 x 10 m 56.57 km
(iii) Time to reach maximum height-474x1414-707s
(iv) x=x = {H COSD)T
y = (a * sin a) * t - 1/2 * g * r ^ 2
9.8 x 3010-2.94
(d ^ 2 * y)/(d * x ^ 2) = g/(v_{0} ^ 2) = 9.8/(10 ^ 2) = 0.098
Using (2) and (3) in (1) we find rho = 305n .
CARLOS, JENNEL
1. the act or quality of acting or moving fast; rapidity. the rate at which something
moves, is done, or acts.
Ans: speed
2. A vibration that propagates as an audible wave of pressure, through a medium
such as a gas, liquid or solid.
Ans: sound
3. is the change in direction of a wavefront at an interface between two
different media so that the wavefront returns into the medium from which it
originated.
Ans: reflection
4. A. is an elementary particle that is a quantum of the electromagnetic
field, including electromagnetic radiation such as light and radio waves, and the
force carrier for the electromagnetic force.
Ans: photon
5. is a scientific concept that is most commonly associated with a state of disorder,
randomness, or uncertainty.
Ans: entropy
6. is a device made of a weight suspended from a pivot so that it can
swing freely. When a pendulum is displaced sideways from its resting,
equilibrium position, it is subject to a restoring force due to gravity that will
accelerate it back toward the equilibrium position.
Ans: pendulum
7. cycle is defined as an ideal reversible closed thermodynamic cycle. Four
successive operations are involved: isothermal expansion, adiabatic expansion,
isothermal compression, and adiabatic compression.
Ans: carnot cycle
8. also called impact, in physics, the sudden, forceful coming together in direct
contact of two bodies, such as, for example, two billiard balls, a golf club and a
ball, a hammer and a nail head, two railroad cars when being coupled together,
or a falling object and a floor.
Ans: collision
9. Branch of physics that deals with the characteristics of fluids at rest,
particularly with the pressure in a fluid or exerted by a fluid (gas or liquid) on an
immersed body.
Ans: hydrostatics
10. also known as angular frequency vector, is a pseudovector representation of
how the angular position or orientation of an object changes with time, i.e. how
quickly an object rotates around an axis of rotation and how fast the axis itself
changes direction.
Ans : angular velocity
CIDRO, JHON ARNOLD
1. Radium has a half like of 8000 years. How much of 40 grams radium would be
left after 16000 years
Solution:
Given data
mass of radium. m-40 grams
half life of radium T1/2 = 8000 years
Required data:
mass remaining after 1600 years=?
Formula:
No.of half lives in 1600 =.1600 =2
8000
answer: mass remaining after 2nd half life = 1/2 x20 =10 gm
Solution:
Given data:
energy. E=5.0 x10^10 J
speed of light c= 2x10^8 m/s
formula:
m=?
E=mc2
m=E =2
C
m= 5.0 x 10^10
( 2x10^8)2
m= 5.0 x10^10
5 x 10^16
m= 10^10 x 10^-16
answer: m= 10^-6 m kg
3. The nucleus of nitrogen contains 36 neutron. Find its charge number it its mass
number 43
Solution:
Given data
No. of neutron. N=36
A=43
Required data: Z=?
charge no.
formula: z= A-N
z=43-36
z= 7
4. Half life of radon is 2.82 days.How much of a 100 gm sample of radon would be
left after 6.64 days
Solution:
Given data
mass of radon = 100 gm
half life = 2.82 days
required data
mass remaining after 6.64 days=?
Formula:
No.of half life = 5.64. =2
2.82
mass remaining after 1st half life = 1/2 x 100
=500
mass remaining after 2nd half life 1/2 x 50gm
answer:25gm
5. What is the PE Stored by a car of weight 1000 N when lifted 50 m straight up?
Solution:
weight of the car w=1000 N
height h=50m
PE=?
Formula:
PE =mgh
w= mg
PE= wh
PE=1000x 50
answer: 50000J
6. A sitar string vibrates of 200 hz.What is the time period of this vibrations
Solution:
Frequency of sitar string f=200
Time Period. T=?
7. A turning fork vibrates 250 times each sec.and produces a ware 1.5m
long.Calculate the velocity of the wave.
solution:
Frequency F=250
wavelength. wl=1.5
v=?
formula: v=fwl
v=250x1.5
answer: v=375
8. The thunder of a cloud is heard 8s after seeing the flash.Find the distance of
the thunder cloud if velocity of sound is 345m/sec
Solution:
velocity of sound. v=345m/sec
time given t=8sec
distance. S=
formula: s=vxt
s=345x8
answer s=2760m
Solution:
length of pendulum l=144cm
gravity t=144. =1.44m
100
s=9.8 m/s^2
t=?
Formula:
t=2m. l
g
t=2x3.142. 1.44
9.8
t= 2x3.142x0.3833
answer: 2.40s
10The potential difference between two points is 120s if an unknown charge is moved
between these points the amount of work si done is 600s.Find the amount of charge.
Solution:
potential difference. v=120v
w=600J
amount of charge. g=?
Formula:
potential difference = work done
charge
= conversion
1qt = 0.946353 L
9.345 qt x 0.946353 L
1 qt
Answer = 8. 844L
Answer = 0.00543
4. David walks 3km north and then toms east and walk 4km. what is the distance and
displacement of the problem?
Distance
= 3km + 4km
= 7 km
Displacement
formula
Displacement = 5km, NE
5. Cassidee walks 1 mile north then turns west and walk 2 miles. What is the distance
and displacement?
Distance = 2mi + 1 mi = 3 mi
Displacement
Formula
6. A Car travel along a straight road 100m east then 50m west. Find the distance and
displacement of a car
50m, w 100m, e
Distance
100m + 50m = 150m
Displacement
100m, E – 50m, W = 50m, E
7. A person walks 4 meter east, then walk a meter north. Determine the distance and
displacement
Distance = 4m + 3m = 7m
Displacement
Formula
8. If 1 pound = 16 ounces, how many pound are in 435 ounces?
Solution:
435 0z = 435 0z x 1 lb
16 0z
= 435 lb
16
Answer = 27. 18 lb
Solution
1 mi x 1760 yrd
1 mi
Solution
234 0z / 1 0z
32,000 ton
= 234
32,000 ton
DAGAMI, RIZALITO
If Michael Jordan has a vertical leap of 1.29 m, then what is his takeoff speed and his
hang time (total time to move upwards to the peak and then return to the ground)
Given: Find:
vf = 0 m/s d = 1.29 m vi = ??
a = -9.8 m/s2
t = ??
vf2 = vi2 + 2*a*d
vi = 5.03 m/s
To find hang time, find the time to the peak and then double it.
vf = vi + a*t
tup = 0.513 s
1. A baseball is popped straight up into the air and has a hang-time of 6.25 s.
Determine the height to which the ball rises before it reaches its peak. (Hint: the
time to rise to the peak is one-half the total hang-time.)
Given: Find:
vf = 0 m/s t = 3.13 s d = ??
2
a = -9.8 m/s
1. (NOTE: the time required to move to the peak of the trajectory is one-half
the total hang time - 3.125 s.)
2. The observation deck of tall skyscraper 370 m above the street. Determine the
time required for a penny to free fall from the deck to the street below.
Given: Find:
75.5 s2 = t2
t = 8.69 s
3. A bullet is moving at a speed of 367 m/s when it embeds into a lump of moist
clay. The bullet penetrates for a distance of 0.0621 m. Determine the
acceleration of the bullet while moving into the clay. (Assume a uniform
acceleration.)
Given: Find:
vf = 0 m/s d = 0.0621 m a = ??
vi = 367 m/s
vf2 = vi2 + 2*a*d
(-134689 m2/s2)/(0.1242 m) = a
a = -1.08*106 m /s2
5. A stone is dropped into a deep well and is heard to hit the water 3.41 s after
being dropped. Determine the depth of the well.
Given: Find:
t = 3.41 s vi = 0 m/s d = ??
a = -9.8 m/s2
d = vi*t + 0.5*a*t2
4. It was once recorded that a Jaguar left skid marks that were 290 m in length.
Assuming that the Jaguar skidded to a stop with a constant acceleration of -
3.90 m/s2, determine the speed of the Jaguar before it began to skid.
Given: Find:
vf = 0 m/s d = 290 m vi = ??
a = -3.90 m/s2
vf2 = vi2 + 2*a*d
vi = 47.6 m /s
6. plane has a takeoff speed of 88.3 m/s and requires 1365 m to reach that speed.
Determine the acceleration of the plane and the time required to reach this
speed.
Given: Find:
(7797 m2/s2)/(2730 m) = a
a = 2.86 m/s2
vf = vi + a*t
t = 30. 8 s
(12544 m2/s2)/(796 m) = a
a = 15.8 m/s2
7. what speed in miles/hr (1 m/s = 2.23 mi/hr) must an object be thrown to reach a
height of 91.5 m (equivalent to one football field)? Assume negligible air
resistance.
Given: Find:
vf = 0 m/s d = 91.5 m vi = ??
2
a = -9.8 m/s
t = ??
First, find speed in units of m/s:
vi = 42.3 m/s
vi = 94.4 mi/hr
6. A car traveling at 22.4 m/s skids to a stop in 2.55 s. Determine the skidding
distance of the car (assume uniform acceleration).
Given: Find:
vf = 0 m/s t = 2.55 s d = ??
vi = 22.4 m/s
d = (vi + vf)/2 *t
d = (11.2 m/s)*2.55 s
d = 28.6 m
DE UNGRIA, WENDELL
Solution: Using the equations of motion, we can find the time the football spends in the
air:
Vy = Vsinθ = 20sin45 = 14.14 m/s
t = Vy/g = 14.14/9.81 = 1.44 s
Then, we can find the horizontal distance traveled:
Vx = Vcosθ = 20cos45 = 14.14 m/s
Dx = Vxt = 14.14 x 1.44 = 20.38 m
Answer: The football will travel 20.38 meters before hitting the ground.
2. A car of mass 1000 kg is traveling at a constant speed of 20 m/s. What is the power
output in watts required to maintain this speed?
3. A baseball player hits a home run, and the ball leaves the bat at an angle of 45
degrees and a speed of 40 m/s. How high does the ball travel above the point where it
was hit before falling back to the ground?
Solution: The maximum height the ball reaches can be calculated using the equation:
H = Vy^2/2g, where Vy is the vertical component of the initial velocity, and g is the
acceleration due to gravity.
Since the initial velocity makes an angle of 45 degrees with the horizontal, we have:
Vy = Vsinθ = 40sin45 = 28.28 m/s
Therefore, the maximum height the ball reaches is: H = (28.28)^2/2 x 9.81 = 41.2
meters
Answer: The ball travels 41.2 meters above the point where it was hit before falling
back to the ground.
4. A car with a mass of 1500 kg is moving at a constant speed of 25 m/s. What is the
kinetic energy of the car?
Solution: The formula for kinetic energy is KE = 0.5mv^2, where m is the mass of the
object and v is its velocity.
5. A car is traveling at a constant speed of 60 km/h for 2 hours. How far has it
traveled?
6. A ball is thrown vertically upward with a velocity of 20 m/s. How high will it go before
it starts falling?
8. A block of ice with a weight of 500 N is sliding down an inclined plane with an angle
of 30 degrees. What is the force acting on the ice parallel to the surface of the inclined
plane?
Solution: We can use the formula, F = m*g*sin(theta) where F is the force acting
parallel to the surface, m is the mass of the block, g is the acceleration due to gravity
and theta is the angle of the inclined plane.
mass = weight/gravity = 500 N/9.8 m/s^2 = 51 kg
Substituting the values to the formula, we get:
F = 51 kg * 9.8 m/s^2 * sin(30 degrees)
F = 249.8 N
Answer: the force acting on the ice parallel to the surface of the inclined plane is 249.8
Newtons.
9. A car moves with an average velocity of 60 km/hr for 2 hours and then with an
average velocity of 40 km/hr for another 3 hours. What is the total distance traveled by
the car?
Solution: To find the total distance, we need to find the distance traveled during each
time period and add them up.
Distance traveled in the first 2 hours:
Distance = Velocity x Time
Distance = 60 km/hr x 2 hr
Distance = 120 km
Distance traveled in the next 3 hours:
Distance = Velocity x Time
Distance = 40 km/hr x 3 hr
Distance = 120 km
Answer: Total distance traveled = 120 km + 120 km = 240 km
10. A roller coaster car with a mass of 2000 kg is at the top of the highest hill of a roller
coaster ride. Its potential energy is 500,000 J. What is the maximum speed the car can
reach when it goes down the hill?
1.A ball is thrown at a velocity of 12ms at an angle of 32o from the horizontal.
What are the ball's horizontal and vertical velocities?
Possible Answers:
Explanation:
The velocity of 12ms can be broken into horizontal and vertical components by using
trigonometry. Think of the figure below, where x and y velocity components of the total
velocity are
shown.
Use the total velocity, the x-component, and the y-component to form a right triangle
below.
Treating 12ms as the hypotenuse, x-component as the leg adjacent, and y-component
as the leg
opposite, you can conclude that the velocities are related through trigonometric
identities.
vy=vsinθ
vx=vcosθ
Plugging in the given values, we can solve for the x and y velocity components.
vy=(12ms)sin(32o)=6.36ms
vx=(12ms)cos(32o)=10.18ms
2. If a 15kg object is subjected to a force of 175N to the east and another force of
1020N to
the west, what is the object’s acceleration?
Possible Answers:
56m/s 2
5.6m/s 2
113m/s 2
1.4m/s 2
Explanation:
First, find the net force by subtracting the opposing forces.
F = 1020N – 175N = 845 N to the west
Next, find acceleration using Newton's second law, F=ma.
a=Fm
a=845N15kg=56ms2(west)
3. Three blocks (in left-to-right order: block A, block B, block C) are pushed along a
frictionless horizontal surface by a 50N force to the right, which acts on block A.
If the masses of the blocks are given by MA=m, MB=3m, and Mc=14m, which of the
following best describes the relationship between the accelerations of the
blocks?
Possible Answers:
aC>aA>aB
aC>aB>aA
aA=aB=aC
aB>aA>aC
aA>aB>Ac
Explanation:
The three blocks must remain in contact as they move, so they will each have the
same velocity and acceleration regardless of their different masses.
So, aA=aB=aC.
4. A 12kg block slides down a 24o incline. If the coefficient of friction between the block
and the incline is 0.2, determine the acceleration of the block as it slides down the
incline.
Possible Answers:
a=0.1ms2
a=3.9ms2
a=2.2ms2
a=5.8ms2
a=7.6ms2
The gravitational force is broken into the x and y components. The net force on the
block in the y-direction is the normal force minus the y component of the gravitational
acceleration (mgcosθ).
FN−mgcosθ=0
Notice that the net y-force is equal to zero to show that the block is not moving
anywhere in the y-direction. Now, we can isolate the normal force.
The net force in the x-direction is mgsinθ−Ffr=ma. We know that the block is
accelerating in the x-direction; therefore the net force is equal to ma.
We can use the friction equations to substitute for the x-direction forces.
Ffr=μkFN=μkmgcosθ
mgsinθ−μkmgcosθ=ma
We can isolate the acceleration and solve using the provided values.
a=gsinθ−μkgcosθ
a=(9.8ms2)sin(24o)−(0.2)(9.8ms2cos(24o)
a=2.2ms2
5. A car of mass 1000kg is initially at rest, and then accelerates
at 2.0ms2 for 14s. What is the kinetic energy of the car at time t=14s?
Possible Answers:
2,000J
39,200J
98,000J
392,000J
20,000J
Explanation:
The first step will be to find the final velocity of the car. We know the acceleration and
time, so we
can find the final velocity using kinematics. The initial velocity is zero, since the car
starts at rest.
vf=vi+at
v=(0ms)+(2ms2)(14s)=28ms
Use this velocity and the mass of the car to solve for the final kinetic energy.
KE=12mv2
KE=12(1000kg)(28ms)2
KE=392,000J
6. A man throws a 2kg ball straight up in the air at a velocity of 20ms. If there is a
constant air resistance force of 0.4N against the motion of the ball, what is the
maximum height of the ball?
Possible Answers:
19.2m
20.83m
36.2m
41.7m
50m
Explanation:
We first need to find the net force acting on the ball during flight. We can then use the
net force and Newton's second law to find the total acceleration on the ball.
Fnet=Fg+Fair
Fnet=mg+Fair
Fnet=(2kg)(−9.8ms2)+0.4N
Fnet=−19.2N
Use this net force to find the acceleration.
Fnet=ma
(−19.2N)=(2kg)a
a=−9.6ms2
From here, there are two ways to solve. One way uses kinematic equations, and the
other uses energy. We will solve using energy.
Total energy must be conserved during the throw, so the initial kinetic energy must
equal the final potential energy (since velocity is zero at the maximum height).
12mv2i+mghi=12mv2f+mghf
12mv2i+mg(0m)=12m(0ms)2+mghf
12mv2i=mghf
Use the given initial velocity to find the final height.
12(2kg)(20ms)2=(2kg)(9.6ms2)(h)
h=12(2kg)(20ms)2(2kg)(9.6ms2)
h=20.83m
7. What is the acceleration of the system shown above? (Assume the table is
frictionless and the mass of the rope connecting blocks is negligible).
Possible Answers:
2.2ms2
14.0ms2
10ms2
7.2ms2
1.8ms2
Explanation:
The force that translates to the entire system is that of gravity acting on the mass
hanging over the ledge.
F=mg=14kg(10m/s2)=140N
140N is the total force acting on the system, which has a mass equal to both blocks
combined (65kg + 14 kg = 79 kg). We can find the acceleration using Newton’s second
law.
A=Fm
A=140N79kg=1.8m/s2
8. A spring is compressed 5cm from equilibrium by a force of 100N. What is the spring
constant for this spring?
Possible Answers:
5Nm
2000Nm
500Nm
200Nm
20Nm
Explanation:
We use Hooke's law equation to relate the force, displacement, and spring
constant:
Fs=−kx
We are given the force and the displacement, allowing us to solve for the spring
constant.
100N=−k(−0.05m)
Note that the displacement is negative, since the spring is compressed. For springs,
compressions represents a negative displacement, while stretching represents a
positive displacement.
k=100N0.05m=2000Nm
9. A mass of weight 20N is suspended vertically from a spring and stretches it 10cm
from equilibrium. What is the energy stored in the spring?
Possible Answers:
4J
2J
1J
0.25J
0.5J
Explanation:
First, we need to solve for the spring constant by using the force on the spring. We can
use Hooke’s Law:
F=−kx
The magnitude of the force on the spring will be equal to the force of gravity on the
mass, which is given to be 20N. The distance the spring it stretched is in the downward
direction, so we must use a negative displacement. Use these values to calculate the
spring constant.
F=(20N)=−k(−0.10m)
K=200Nm
Next, use the spring energy equation with the displacement and spring constant to
solve for the energy stored in the spring.
E=12kx2
E=12(200Nm)(−0.10m)2
E=1J
10. A ball of mass 1kg on a string is rotating about the center axis with a velocity 3ms.
If the tension force in the string is 15N, what is the radius of the circular path of the
ball?
Possible Answers:
1.0m
0.4m
0.8m
0.6m
0.2m
Explanation:
We use the equation for centripetal force to find the radius:
Fc=mv2r
Since the string ties the ball to the axis, the force of tension will be equal to the
centripetal force.
FT=15N=mv2r
Use the given mass and velocity to solve for the radius.
15N=(1kg)(3ms)2r
R=9m2kgs215N=0.6m
FERRERAS, LANCE
1. What is distance?
Answer: Distance is the total movement of an object, irrespective of the direction.
2. What is displacement?
Answer: Displacement is the shortest distance between the initial and final position of
an object, with a specified direction.
4. If a person walks around a circular track and ends up at the starting point,
what is the displacement?
Answer: The displacement is zero because the initial and final positions are the same.
8. If a person walks 10 meters east and then 10 meters west, what is the total
distance covered and the total displacement?
Answer: Distance = 20 meters; Displacement = 0 meters.
PROBLEM SOLVING
FORCE VECTOR, DISTANCE AND DISPLACEMENT, UNIT OF MEASUREMENTS,
Force Vector
Problem 1:
A 1000 N force is applied at an angle of 30 degrees above the horizontal. Another 800
N force is applied at an angle of 45 degrees below the horizontal. Find the resultant
force.
Solution
To find the resultant force, you can break down each force into its horizontal and
vertical components. For the 1000 N force:
Horizontal component: 1000 N * cos(30 degrees)
Vertical component: 1000 N * sin(30 degrees)
For the 800 N force:
Horizontal component: 800 N * cos(45 degrees)
Vertical component: -800 N * sin(45 degrees) (negative because it’s below the
horizontal)
Now, add the horizontal and vertical components separately:
Horizontal component of resultant force = (1000 N * cos(30 degrees)) + (800 N *
cos(45 degrees))
Vertical component of resultant force = (1000 N * sin(30 degrees)) + (-800 N * sin(45
degrees))
Finally, you can find the magnitude and direction of the resultant force using.
Problem 2:
A box is placed on an inclined plane that makes a 20-degree angle with the horizontal.
If the weight of the box is 500 N, find the force of gravity acting on the box parallel to
the incline and perpendicular to the incline.
Solution
The weight of the box acts vertically downward and can be split into two components:
one parallel to the incline and one perpendicular to the incline.
Force parallel to the incline (Fp) = Weight
*sin(20 degrees)
Displacement: In this case, your displacement is 50 meters to the east from your initial
position (point A) Because it measures the change in your position from the start to the
end, regardless of the path taken.
Unit of measurement
Problem 5: Projectile Motion
Problem: A ball is kicked with an initial velocity of 20 meters per second (m/s) at an
angle of 30 degrees above the horizontal. Calculate the maximum height it reaches.
Solution:
1. Calculate the vertical component of the initial velocity: V_vertical = V_initial *
sin(0) = 20 m/s sin(30°)≈ 10 m/s *
2. Use the kinematic equation for maximum height: H_max = (V_vertical^2)/(2* g)
where g is the acceleration due to gravity (approximately 9.81 m/s^2)
3. 3. Plug in the values: H_max=(10 m/s)^2/(2* 9.81 m/s^2) 5.1 meters
Problem 6 : Simple Pendulum
Problem: A pendulum has a length of 1.5 meters. Calculate its period, which is the time
it takes to complete one full swing.
Solution:
1. Use the formula for the period of a simple pendulum:
T = 2π* √(L/g)
Where L is the length of the pendulum and g is the acceleration due to gravity.
2. Plug in the values:
T= 2* √(1.5 m/9.81 m/s^2)
3.
Calculate the period:
Solution:
To multiply numbers in scientific notation, you can multiply their significant figures and
add their exponents.
(2.5 x 10^3)* (3.0 x 10^4) = (2.5 * 3.0) * 10^(3+4)=7.5 * 10^7
So, the result is 7.5 x 10^7 meters- seconds, which can also be written as 75,000,000
meters-seconds.
Solution:
The total distance covered is simply the sum of the distances traveled in each
direction: 10 km + 10 km = 20 kilometers. However, the displacement is zero since the
cyclist ends up back at the starting point.
Problem 10: Displacement with a Turn Problem: A person walks 5 meters north, then
turns and walks 5 meters east. Calculate the displacement.
Solution:
To calculate the displacement, we can use the Pythagorean theorem. The person’s
displacement is the hypotenuse of a right triangle with legs of 5 meters each:
Displacement = √((5 m)^2 + (5 m)^2) = √(25 m^2 + 25 m^2) = √(50 m^2) ≈ 7.07 meters
at an angle of 45 degrees northeast.
FRANCISCO, JACK TRACY
4. How can you calculate displacement when you have initial and final positions?
Answer: Displacement (Δx) can be calculated as the final position (xf) minus the initial
position (xi), i.e., Δx = xf - xi.
7. Can an object have a non-zero displacement while traveling a total distance of zero?
Answer: Yes, an object can have a non-zero displacement if it changes its position
and then returns to its initial position. In this case, the total distance is zero.
A professor going to work first walks 500 m along the campus wall, then enters the
campus and goes 100 m perpendicularly to the wall towards his building, after that
takes an elevator and mounts 10 m up to his office. The trip takes 10 minutes.
Calculate the displacement, the distance between the initial and final points, the
average velocity and the average speed.
500 m 1
0 0
d ,d 3 y
100 m
10 m
2
x
Solution: The total trajectory can be represented by three vectors going from 0 to 1,
then from 1 to 2, then from 2 to 3. The displacement is the vector sum of the three
displacement vectors:
The distance 𝑑 between the initial and final points is the magnitude of the
displacement 𝐝:
∆𝐫 𝐝 (100,500,10)
𝐯= = = = (0.167, 0.833, 0.017) m/s.
∆𝑡 10 × 60
∆
𝑡
𝑑 510
𝑣 = |𝐯| = = = 0.85 m/s.
10 × 60
∆
𝑡
𝑤 610
𝑠= = = 1.02 m/s.
10 × 60
∆
𝑡
2. A 2D walker
A walker goes 1000 m the direction 30 degrees North of East, then 2000 m in the
South- West direction. The trip takes 30 minutes.
Find the displacement, way length, average velocity and average speed.
North y
r01 1
0 30° 45°
O East
West x
d ,d r12
South
2
where
√3 1
𝒓01 = (𝑟01,𝑥, 𝑟01,𝑦) = (𝑟01 cos 30°, 𝑟01 sin 30°) = , 1000 ) = (500√3, 500) m
(1000 2 2
and
𝒓12 = (𝑟12,𝑥, 𝑟12,𝑦) = (−𝑟12 cos 45°, −𝑟12 sin 45°) = (−2000
√2 √2
, − 2000 )
2 2
= (−1000√2, − 1000√2, ) m.
Better is to write
𝒓12 = (𝑟12,𝑥, 𝑟12,𝑦) = (𝑟12 cos 125°, 𝑟12 sin 125°) = (2000 (− √2 ) , 2000 (− √2 ))
2 2
= (−1000√2, − 1000√2, ) m
The velocity:
𝑑 1066
𝑣 = |𝐯| = = = 0.59 m/s.
30 × 60
∆
𝑡
𝑤 3000
𝑠= = = 1.67 m/s > 𝑣.
30 × 60
∆
𝑡
3. Motion with constant acceleration
A car started moving from rest with a constant acceleration. At some moment of time,
it covered the distance 𝑥 and reached the speed 𝑣. Find the acceleration and the time.
Solution. The formulas for the motion with constant acceleration read
1
𝑣 = 𝑎𝑡, 𝑥= 𝑎𝑡2,
2
where we have taken into account that the motion starts from rest (all initial values
are zero). If 𝑣 and 𝑥 are given, this is a system of two equations with the unknowns
𝑎 and 𝑡. This system of equations can be solved in different ways.
First method. For instance, one can express the time from the first equation, 𝑡 = 𝑣/𝑎,
and substitute it to the second equation,
1𝑣 2
𝑥= 2= 𝑣 .
𝑎( ) 2𝑎
2 𝑎
From this single equation for 𝑎 one
finds
2
𝑎=𝑣 .
2𝑥
Now, one finds the time
as
𝑣 𝑣 2𝑥
𝑡= = = .
𝑎 𝑣 /(2𝑥)
2
𝑣
1 1
𝑥= 𝑎𝑡 × 𝑡 = 𝑣𝑡.
2 2
After that one
finds
2𝑥
𝑡= ,
𝑣
and,
further,
� 2
�=
𝑣 = 𝑣 .
𝑎=
𝑡 2𝑥/𝑣 2𝑥
4. A car trip (1D motion)
A car starts from the place with an acceleration 2 m/𝑠2 and is accelerating during
10 seconds, then travels with the same speed for 30 seconds, then decelerates
at the rate 3 m/𝑠2 until stopping. Show the graph 𝑣(𝑡). Calculate the total time of
the trip and the distance covered in each interval and the total distance covered
by two methods: 1) Calculation of the area under the line 𝑣(𝑡); 2) Using the
formula for the distance in the motion with constant acceleration.
v v1
1 2 3
0 t1 t2 t3 t
𝑣1 = 𝑎1𝑡1.
Interval 2: 𝑣 = 𝑣1.
𝑡2 = 𝑡1 + ∆𝑡2 = 10 + 30 = 40 𝑠.
(this is the velocity formula with shifted time as the motion starts at 𝑡 = 𝑡2 rather than
at
𝑡 = 0). At the end of the motion the car stops that is described by the instance of the
formula above with 𝑣 = 0, that is,
0 = 𝑣1 + 𝑎3(𝑡3 − 𝑡2)
𝑣1 𝑎1𝑡1 𝑎1
∆𝑡3 ≡ 𝑡3 − 𝑡2 = − =− =− 𝑡1
𝑎 3 𝑎3 𝑎3
and,
further,
𝑎1
𝑡 1.
𝑡3 = 𝑡2 + ∆𝑡3 = 𝑡1 + ∆𝑡2 + ∆𝑡3 = 𝑡1 + ∆𝑡2 −
𝑎 3
This is the analytical or symbolic or algebraic answer or formula for the total time.
(This result will not be used, however). In this formula, the result is expressed
through the quantities given in the formulation of the problem (this has to be
checked each time before submitting the solution for grading!). Now, substituting
given numbers, one obtains
2 20
𝑡3 = 10 + 30 − 10 = 10 + 30 + = 46.7 𝑠.
−3 3
The preparatory work done, let us now find the total distance covered. Using the first
method, we find it as the area under the curve 𝑣(𝑡) that consists of two triangles and
one rectangle, see the figure. The parameters of them have been calculated above.
So we write
1 1
∆𝑥 = ∆𝑥1 + ∆𝑥2 + ∆𝑥3 = 𝑡1𝑣1 + ∆𝑡2𝑣1 + ∆𝑡3𝑣1.
2 2
Here we must substitute the expressions for the quantities that are not given in the
problem formulation: 𝑣1 and ∆𝑡3. We prefer not to factor 𝑣1 to keep the contributions
of each interval separately. The result reads
2 1 1 𝑎1
∆𝑥 = 𝑎1𝑡1 + ∆𝑡2𝑎1𝑡1 + (− 𝑡1) 𝑎1𝑡1
2 2 𝑎3
or, finally,
1 𝑡2 + ∆𝑡 1 𝑎12
∆𝑥 = 𝑎 𝑎 − 𝑡 2.
𝑡
2 1 2𝑎 1
2 1 1 1 3
This is our symbolic answer for the distances covered in the motion.
Substituting the numerical values from the problem’s formulation, one obtains
1 1 22
∆𝑥 = 2 × 102 + 30 × 2 × 10 − 102 = 100 + 600 + 66.7 = 766.7𝑚.
2 2 (−3)
Now, let us find the total distance covered using the formula for the displacement in
the motion with a constant acceleration
1 2
∆𝑥 ≡ 𝑥 − 𝑥0 = 𝑣0∆𝑡 + 𝑎(∆𝑡)
2
in the form appropriate to each of the motion intervals. One has
1 1
∆𝑥 = ∆𝑥 + ∆𝑥 + ∆𝑥 =𝑎 𝑡 + 𝑣 ∆𝑡 + [𝑣 ∆𝑡 + 𝑎 (∆𝑡 )2]
2
1 2 3 2 1 1 1 2 1 3 2 3 3
1 2 1
= 𝑎1𝑡1 + 𝑣1∆𝑡2 + [𝑣1 + 𝑎3∆𝑡3] ∆𝑡3.
2 2
Substituting here the expressions for 𝑣1 and ∆𝑡3, one obtains
∆𝑥 = 1 2
1 𝑎1 𝑎1
𝑎1𝑡1 + ∆𝑡2𝑎1𝑡1 + [𝑎1𝑡1 + 𝑎3 (− 𝑡1)] (− 𝑡1)
2 2 𝑎3 𝑎 3
1 2 𝑎1
1
= 𝑎1𝑡1 + ∆𝑡2𝑎1𝑡1 + [𝑎1𝑡1 − 𝑎1𝑡1] (− 𝑡1)
2 2 𝑎3
1 2
= 1 𝑎1
𝑎1𝑡1 + ∆𝑡2𝑎1𝑡1 + 𝑎1𝑡1 (− 𝑡1)
2 2 𝑎3
1
𝑡2 + ∆𝑡 𝑎 − 1 𝑎1 𝑡 2
2
=𝑎
𝑡
1 1 2 1 2𝑎 1
2 1 3
that coincides with the result obtained by the first method (the red formula).
A rocket starts vertically up and moves with the acceleration 20 𝑚/𝑠2 during 20
seconds. Then it continues its motion ballistically. Find the maximal height reached
and the corresponding time. Find the time of hitting the ground and the
corresponding speed.
Solution. First, we introduce the notations: the duration of the first stage (powered
motion)
𝑡1 = 20 𝑠, the acceleration in the first stage 𝑎 = 20 𝑚/𝑠2. The initial velocity is zero.
We choose the origin of time 𝑡 = 0 and put the origin of 𝑧-axis (directed up) at zero, so
that the initial conditions are 𝑧0 = 0 and 𝑣0 = 0. The formulas for the motion with a
constant acceleration at the first stage are
1
𝑣 = 𝑎𝑡, 𝑧= 𝑎𝑡2.
2
At the end of the first stage, 𝑡 = 𝑡1, the velocity and the height read
1 2
𝑣1 = 𝑎𝑡1, 𝑧1 = 𝑎𝑡1 .
2
These are the initial conditions for the motion on the second stage. The formulas for
the motion with the constant acceleration −𝑔 on the second (ballistic) stage are
𝑣 = 𝑣 − 𝑔 (𝑡 − 𝑡 ) 𝑧 = 𝑧 + 𝑣 (𝑡 − 𝑡 ) 1 ( )2.
1 1 1 1 − 𝑔 𝑡 − 𝑡1
2
1
Note that the second stage begins at 𝑡 = 𝑡1, thus we use the formulas with shifted
time. The highest point can be found from the condition 𝑣 = 0 that yields the equation
for the time at which the maximal height is reached:
0 = 𝑣1 − 𝑔(𝑡𝑚𝑎𝑥 − 𝑡1).
The solution
is
𝑣1 𝑎𝑡1 𝑎
𝑡𝑚𝑎𝑥 − 𝑡1 = = = 𝑡1
𝑔 𝑔 𝑔
and,
finally,
𝑎 + 1) 𝑡1.
𝑎
𝑡𝑚𝑎𝑥 = 𝑡1 + 𝑡1 = ( 𝑔
𝑔
Substituting the numbers, one obtains
20
𝑡𝑚𝑎𝑥 = ( + 1) 20 = 60.8 𝑠.
9.8
1
𝑧𝑚𝑎𝑥 = 𝑧1 + 𝑣1 (𝑡𝑚𝑎 − 𝑡1) − 2 𝑔(𝑡𝑚𝑎 − 𝑡1)2.
𝑥 𝑥
Substituting here the quantities found above and simplifying, one obtains
1 2 𝑎 1 𝑎 2
𝑧𝑚𝑎𝑥 = 𝑎𝑡1 + 𝑎𝑡1 𝑡1 − 𝑔 𝑔 𝑡1)
( 2 𝑔 2
1
= 𝑎𝑡2 + 𝑎 2 1
2 1 2 𝑡 − 𝑎2 𝑡2
𝑔 1 2 𝑔 1
1 1 𝑎
= 𝑎𝑡2 + 1 (1 + ) 𝑎𝑡1.
2
𝑡2 =
2 1 𝑎
2
2 𝑔
2 𝑔 1
At the third stage, the rocket falls with the acceleration −𝑔 from the height 𝑧𝑚𝑎𝑥. The
formulas for its velocity and height at this stage are
𝑣 = −𝑔(𝑡 − 𝑡 ), 𝑧=𝑧 1 ( )2
𝑚𝑎𝑥 𝑚𝑎𝑥 − 2 𝑡 − 𝑡𝑚𝑎𝑥
𝑔
(also formulas with shifted time). The final time (of hitting the ground) 𝑡𝑓 is determined
by
𝑧 = 0. This gives the equation (an instance of the general formula)
1 2
2𝑧𝑚𝑎𝑥 21 𝑎 𝑎 𝑎
=√ 2 )
𝑡𝑓 − 𝑡𝑚𝑎𝑥 = √ (1 + ) 𝑎𝑡1 = √(1 + 𝑔 𝑡1.
𝑔 𝑔 𝑔
2 𝑔
This will be needed to find the final velocity. For 𝑡𝑓 itself one obtains
𝑡𝑓 = 𝑡𝑚𝑎𝑥 + √(1 𝑎 𝑎 𝑎
+ 𝑎 𝑡1 = ( + 1) 𝑡1 + √(1 + 𝑎 𝑡1.
) )
𝑔 𝑔 𝑔 𝑔 𝑔
9.8
(Is it obvious that it takes a longer time for the rocket to reach the highest point than to
fall back to the initial level? To understand this, sketch the function 𝑧(𝑡).)
Now, the final velocity can be found from the velocity formula:
Solution. First, we define the coordinate axes and introduce missing notations. The
origin of the coordinate system is at the server’s position, 𝑧-axis up and 𝑥-axis to the
right. The initial height (serve height) 𝑧0 = 2.5 𝑚, the height of the net 𝑧1 = 0.9 𝑚, the
height of the ground (the reference height) 0 𝑚, distance server-net 𝑥1 = 15 𝑚. Find
𝑣0𝑥.
1 2
𝑥 = 𝑣0𝑥 𝑡 𝑧 = 𝑧0 − 𝑔𝑡 .
, 2
The instance of these general formulas corresponding to the ball passing just above
the net
reads
𝑥 =𝑣
𝑡 , 1 2
1 0𝑥 1 𝑧1 = 𝑧0 − 𝑔𝑡1 .
This is a system of two equations with two unknowns:2 𝑣0𝑥 and 𝑡1. The second equation
is
autonomous (contains only one unknown), so it can be solve to give
2(𝑧0 − 𝑧1)
𝑡1 = √ .
𝑔
Then, from the first equation one finds
𝑥1 𝑔
= 𝑥1√ .
𝑣0𝑥 = 2 (𝑧0 − 𝑧1)
𝑡1
9.8 𝑚
𝑣0𝑥 = 15√ = 26.3 .
2(2.5 − 0.9) 𝑠
Now we can find the distance from the server at which the ball lands. We use the
instances of the general formulas above corresponding to the ball hitting the ground:
𝑥 =𝑣 𝑡 , 1 2
2 0𝑥 2 0 = z = 𝑧0 − 𝑔𝑡2 .
One finds 𝑡2 from the second 2
equation:
2𝑧0
𝑡2 = √ .
𝑔
From this formula, one can find the numerical value of 𝑡2 that is the total time of
the motion. Substituting the formula for 𝑡2 into the first equation, one obtains
𝑔 √ 2𝑧0 = 𝑥1√ 𝑧0 .
𝑥2
𝑥1√= 𝑣0𝑥𝑡2 = −𝑧 ) 𝑔 𝑧 −𝑧
2(𝑧
0 1 0 1
𝑥2 2.5
= 15√ 2.5 − = 18.75 𝑚.
0.9
Now 𝑥2 − 𝑥1 = 18.75 − 15 = 3.75 𝑚 that is well below 7 𝑚. Thus, the ball is “good”.
Solution: First, we must introduce missing notations: the height of the copter ℎ = 78
𝑘𝑚 1000
𝑚, the speed of the copter 𝑣 = 215 = 215 × = 59.9𝑚/𝑠, the speed of the
car 𝑢 =
𝑘𝑚 155 ℎ 3600
155 = = 43.1𝑚/𝑠. Find the angle 𝜃.
ℎ 3.6
There are two solutions to this problem, in the laboratory frame and in the moving
frame of the car.
Solution in the laboratory frame. Put the origin of the coordinate system on the
ground below the copter. The initial 𝑥-coordinate of the car(when the package is
dropped) is 𝑥𝑐,0. If it is found, then the angle 𝜃 can be expressed as
tan 𝜃 =
ℎ
.
𝑥𝑐,
0
The formulas for the motion of the package and the car have the form
1 2
𝑧𝑝 = ℎ − 𝑔𝑡 , 𝑥𝑝 = 𝑣𝑡, 𝑥𝑐 = 𝑥𝑐,0 + 𝑢𝑡.
2
When the package lands into the car, the following conditions are fulfilled:
𝑧𝑝 = 0, 𝑥𝑝 = 𝑥𝑐.
Substituting these into the general equations, one obtains their instance
1
0=ℎ− 𝑔𝑡2
2 , 𝑣𝑡 = 𝑥𝑐,0 + 𝑢𝑡.
This is a system of two equations with two unknowns. The first equation is
autonomous and yields the fall time
2ℎ
𝑡 = 𝑡𝑓 = √ .
𝑔
Substituting this into the second equation, one obtains
2ℎ
𝑥𝑐,0 = (𝑣 − 𝑢)𝑡𝑓 = (𝑣 − 𝑢)√ .
𝑔
Now for the angle one obtains
ℎ 𝑔 1 𝑔ℎ
𝜃 = arctan ( √ ) = arctan ( √ ).
𝑣 − 𝑢 2ℎ 𝑣−𝑢 2
1 9.8 ×
𝜃 = arctan ( 78 ) = arctan(1.16) = 49°.
√
(59.9 − 43.1) 2
Solution in the moving frame (frame of the car). The absolute velocity of the copter
can be represented as
𝑣 = 𝑣′ + 𝑢,
where 𝑣′ is the relative velocity of the copter with respect to the car, 𝑣′ = 𝑣 − 𝑢. The
origin of the coordinate axes in the moving frame, 𝑂′, is moving to the right with the
velocity of
the car 𝑢. At 𝑡 = 0 the origins of the laboratory and moving frames coincide, 𝑂′ = 𝑂.
Thus the relation between the 𝑥-coordinate (absolute frame) and 𝑥′-coordinate
(moving frame) is
𝑥 = 𝑥′ + 𝑢𝑡
or,
conversely,
𝑥′ = 𝑥 − 𝑢𝑡
The formulas for the motion of the package in this frame have the form
1 2 ′ ′
𝑧𝑝 = ℎ − 𝑔𝑡 , 𝑥 𝑝 = 𝑣 𝑡.
2
As for the car, it is at rest in its own frame:
𝑥′𝑐(𝑡) = 𝑥𝑐,0.
2ℎ
𝑡𝑓 = √ ,
𝑔
and substitutes it into the
condition:
𝑥′𝑝(𝑡𝑓) = 𝑥′𝑐(𝑡𝑓)
or
𝑝(𝑡𝑓) = 𝑣 𝑡𝑓
′
𝑥′ 2ℎ ) = 𝑥𝑐,0.
(
= 𝑣−𝑢 )√ = 𝑥 (𝑡
′
𝑔 𝑐
𝑓
The result for 𝑥𝑐,0 coincides with that obtained by the first method:
2ℎ
𝑥𝑐,0 = (𝑣 − 𝑢)√ .
𝑔
The
n
ℎ ℎ 𝑔 1
tan 𝜃 = = √ =
𝑥𝑐, 𝑣−𝑢 𝑣 − 𝑢 𝑔ℎ
√ 2,
0
2ℎ
A cannon launches missiles with the initial speed 𝑣0. Find the targeting angles 𝜃 to hit
the target at the distance 𝑑 at the same height as the cannon.
Solution. The formulas for the projectile motion have the form
1 2
𝑧 = 𝑣0𝑧𝑡 − 𝑔𝑡 , 𝑥 = 𝑣0𝑥𝑡.
2
The origin of the coordinate system is put at the location of the cannon, thus 𝑥0 =
𝑧0 = 0. The distance between the cannon and the landing point is defined by the
fall time (or final time or flight time) 𝑡𝑓:
𝑑 = 𝑣0𝑥𝑡𝑓.
1 2 1
0 = 𝑣0𝑧𝑡𝑓 − 𝑔𝑡𝑓 = 𝑡𝑓 (𝑣0𝑧 − 𝑔𝑡𝑓).
2 2
The first solution to this equation, 𝑡𝑓 = 0, corresponds to the beginning of the motion
and should be discarded. The landing time nullifies the expression in the brackets,
1
𝑣0𝑧 − 𝑔𝑡𝑓 = 0,
2
wherefrom
2𝑣0𝑧
𝑡𝑓 = .
𝑔
Now
𝑑 = 𝑣0𝑥𝑡𝑓 = 2𝑣0𝑥𝑣
0𝑧
.
𝑔
so that
2𝑣2 sin 𝜃 cos 𝜃 𝑣2 sin 2𝜃
0 0
𝑑 = 𝑔 = 𝑔 ,
where the trigonometric identity sin 2𝜃 ≡ 2 sin 𝜃 cos 𝜃 was used. As the maximal
value of the sine function is 1 and it is reached for the argument equal to 90°, one
can see that 𝑑 reaches its maximum for 𝜃 = 45°. One can rewrite
2
𝑑=𝑑 sin 2𝜃 , = 𝑣0 .
𝑚𝑎 𝑚𝑎 𝑔
𝑥 𝑑 𝑥
This is an equation for 𝜃 if 𝑑 has a prescribed value (the distance to the target). For
the distance to the target 𝑑 > 𝑑𝑚𝑎𝑥 the target cannot be hit. For 𝑑 < 𝑑𝑚𝑎𝑥 the target
can be hit in two different ways using two values of the targeting angle that satisfy
𝑑
sin 2𝜃 = 𝑑 .
𝑚𝑎
𝑥
These solutions are
2𝜃1 = arcsin 𝑑 𝑑
and 2𝜃2 = π − arcsin ,
𝑑 𝑚𝑎 𝑚𝑎𝑥
𝑥 𝑑
that is,
1 𝑑 π 1 𝑑
𝜃1 = arcsin and 𝜃2 = − arcsin 𝑚𝑎𝑥.
2 𝑚𝑎 2 2
𝑑 𝑑
𝑥
The second solution is in radians, and 𝜋 radians corresponds to 180°. For instance,
for
𝑑 1 1
= one has arcsin2 = 30° and 𝜃1 = 15° and 𝜃2 = 90° − 15° = 75°.
2
𝑑𝑚
𝑎𝑥 Equation: sin a
sin
1 Solutions:
a<1
1 arcsina
0 1 2 arcsina
cos
One can check algebraically that the second expression is also a solution of the
equation:
The general formulas for the projectile motion have the form
1 2
𝑧 = 𝑣0𝑧𝑡 − 𝑔𝑡 , 𝑥 = 𝑣0𝑥𝑡.
2
The origin of the coordinate system is put at the location of the cannon, thus 𝑥0 = 𝑧0 =
0.
1 2
ℎ = 𝑣0𝑧𝑡𝑓 − 𝑔𝑡𝑓 ,
𝑑 = 𝑣0𝑥𝑡𝑓.
2
From here, one finds the components of the initial velocity:
𝑑 ℎ + 1𝑔𝑡2
𝑣0𝑥 = , 𝑣0 = 2 𝑓
𝑡𝑓 .
𝑡𝑓
Now
2
√𝑑2 + (ℎ + 1𝑔𝑡2)
𝑣 = √𝑣2 + 𝑣2 = 2 𝑓
0 0𝑥 0𝑧 𝑡𝑓
We put the origin of the coordinate system at the foot of the “cliff” (below the end of
the takeoff ramp at the level of the roofs of the standing cars). The formulas for the
motion with a constant acceleration have the form
1 2
𝑧 = ℎ + 𝑣0𝑧𝑡 − 𝑔𝑡 , 𝑥 = 𝑣0𝑥𝑡.
2
When the jumping car clears the roof of the last standing car, one has (an instance of
the formulas above)
1 2
0 = ℎ + 𝑣0𝑧𝑡𝑓 − 𝑔𝑡𝑓 , 𝑑 = 𝑣0𝑥𝑡𝑓. (1)
2
(a) In this case 𝑣0𝑧 = 0 and 𝑣0𝑥 = 𝑣0. From the first equation one obtains
2ℎ
𝑡𝑓 = √ .
𝑔
Substituting this into the second equation, one obtains
𝑣0 = 𝑑 𝑔
𝑡 = 𝑑√ .
𝑓 2ℎ
9.8
𝑣0 = 20√ = 36 𝑚/𝑠.
2 × 1.5
so that the general formulas at the clearing point, (1), take the form
1 2
0 = ℎ + 𝑣0 sin 𝜃 𝑡𝑓 − 𝑔𝑡𝑓 ,
𝑑 = 𝑣0 cos 𝜃 𝑡𝑓.
2
This is again a system of two equations with two unknowns: 𝑣0 and 𝑡𝑓. However, it is
inconvenient to find 𝑡𝑓 from the first equation, as above, because here one needs to
sole a full quadratic equation. Thus we apply a slightly different method. Since we
do not need 𝑡𝑓, we can eliminate if from the second equation [ 𝑡𝑓 = 𝑑/(𝑣0 cos 𝜃) ] and
substitute it into the first equation that yields
0 = ℎ + 𝑣0 sin 𝑑 − 1 𝑔 ( 𝑑 ) 2.
𝜃 2 𝑣0 cos 𝜃
𝑣0 cos
𝜃
After simplification, one obtains the equation for the car’s speed
𝑔𝑑2 1
0 = ℎ + 𝑑 tan 𝜃 −
2 cos2 𝜃 𝑣02
that is a quadratic equation without the linear term
Its solution
reads
𝑑 𝑔
𝑣0 = √ .
cos 𝜃 2 ( ℎ + 𝑑 tan 𝜃)
For 𝜃 = 0, this formula simplifies to the solution obtained in (a). For small 𝜃, one can
use
1
sin 𝜃 ≅ 𝜃, tan 𝜃 ≅ 𝜃, cos 𝜃 ≅ 1 𝜃2 ≅ 1,
− 2
(𝜃 in radians) so that the value of 𝑣0 decreases with 𝜃 because of the tan 𝜃 term in the
denominator. We have the angle, in radians,
2𝜋
𝜃 = 10° = 0.175 ≪ 1,
360°
so that the angle is, indeed, small, and one can use the formulas for the small angles
𝑣0 ≅ 20√
( 9. 9.
= 20√ = 19.80 𝑚/𝑠.
8 ) ( 8 )
2 1.5 + 20 × 0.175 2 1.5 + 3.5
Using the full expression yields
20 9.8
𝑣0 = √ = 20.05 𝑚/𝑠.
(2 1.5 + 20 tan )
cos 10°
10°
This is a serious decrease of the minimal speed in comparison to the case 𝜃 =
0. The reason is that the small tan 𝜃 is multiplied by the large 𝑑.
= 59.63° ≈ 60°.
GERSALINA, RENZ JOHN
Answer:
Given :
Force mg = 30 N ; height = 10 m
Work done to lift a load W = ?
W = F.S (or) mgh
= 30x10
W = 300 J
Ans: 300J
2. A ball with a velocity of 5 m s-1 impinges at angle of 60˚ with the vertical on a
smooth horizontal plane. If the coefficient of restitution is 0.5, find the velocity and
direction after the impact.
Answer
The impluse on the ball acts perpendicular to the smooth plane.
v = 3.3 ms-1
3. A bob of mass m is attached to one end of the rod of negligible mass and
length r, the other end of which is pivoted freely at a fixed center O as shown in
the figure. What initial speed must be given to the object to reach the top of the
circle? (Hint: Use law of conservation of energy). Is this speed less or greater
than speed obtained in the section 4.2.9?
AC = OA-OC = r - rcosθ
minimum velocity = at vL = √[5gr]
Answer:
Momentium is censerved in both × and y direction.
In x - direction
MBVB = O + MAVA' cosɸ ...(l)
In y - direction
O = MBVB' - MAVA'sinɸ ...(2)
(2)/(1) tanɸ = VB’/VB = 1/2
tan ɸ = ½
ɸ = 26.6° (or) 26° 36' [1° = 60']
Answer
Mass of the bullet m1 = 20 g = 0.02 kg.
Mass of the pendulum m2 = 5 kg
Centre of mass of pendulum rises to a height = h = 10 cm = 0.1 m
Speed of the bullet = u1
Pendulum is at rest .:. u2 = 0
Common velocity of the bullet and the pendulum after the bullet is embeded into
the object = v
From II equation of motion
v = √[2gh] = √[2x9.8x0.1] = √[1.96] = 1.4 ms-1
Substitute the value of v in equation (1)
1.4 = 0.02 u1/5.02
u1 = 5.02x1.4 / 0.02
u1 = 351.4 ms-1
Solution:
constant = k
Un-stretched length = l
Angular velocity = ω
7.Two objects of masses 2 kg and 4 kg are moving with the same momentum of
20 kg m s-1.
Solution
8.Even though both are having the same momentum, the kinetic energy of both
masses is not the same. The kinetic energy of the heavier object has lesser
kinetic energy than smaller mass. It is because the kinetic energy is inversely
proportional to the mass (KE ∝ 1/m) for a given momentum.
b. As the momentum, p = mv, the two objects will not have same speed.
c. Suppose the spring is compressed to the same 25 cm, calculate the potential
energy stored and also the work done by the spring force during compression.
(The spring constant, k = 0.1 N m-1).
Solution
c. During compression also the potential energy stored in the object is the same.
9.A body of mass 100 kg is lifted to a height 10 m from the ground in two different
ways as shown in the figure. What is the work done by the gravity in both the
cases? Why is it easier to take the object through a ramp?
Solution
m = 100 kg, h = 10 m
Along path (1):
The minimum force F1 required to move the object to the height of 10 m should
be equal to the gravitational force, F1 mg = 100 x 10 = 1000 N
W = Fh = 1000 x 10 = 10,000 J
In the case of the ramp, the minimum force F2 that we apply on the object to take
it up is not equal to mg, it is rather equal to mg sinθ .(mg sin < mg) .
The work done on the object along path (2) is, W = F2 l = 500 × 20 = 10,000 J
Since the gravitational force is a conservative force, the work done by gravity on
the object is independent of the path taken.
In both the paths the work done by the gravitational force is 10,000 J
Along path (1): more force needs to be applied against gravity to cover lesser
distance .
Along path (2): lesser force needs to be applied against the gravity to cover more
distance.
As the force needs to be applied along the ramp is less, it is easier to move the
object along the ramp.
10.A lighter particle moving with a speed of 10 m s-1 collides with an object of
double its mass moving in the same direction with half its speed. Assume that the
collision is a one dimensional elastic collision. What will be the speed of both
particles after the collision?
Solution
Let the mass of the fi rst body be m which moves with an initial velocity, u1 = 10
m s-1.
Then, the fi nal velocities of the bodies can be calculated from the equation
(4.53) and equation (4.54)
As the two speeds v1and v2 are positive, they move in the same direction with
the velocities, 3.33 m s−1 and 8.33 m s−1 respectively.
GESTO, MATHEW
1. Problem: A car accelerates from rest at 2 m/s2 for 5 seconds. What is its final
velocity?
Solution: Use the equation: v=u+at
v = 0 + (2m/s2)(5s) = 10m/s
2. Problem: An object is moving with a constant speed of 20 m/s. How far will it
travel in
10 seconds?
Solution: Use the formula: d = vt
:
d = (20m/s)(108) = 200meters
4. Problem: An object moves with a velocity of 30 m/s for 5 seconds and then
accelerates at -2 m/s2 for the next 3 seconds. Calculate the total distance
traveled.
Solution: Split the problem into two parts. First, calculate the distance during the
initial 5
seconds:
d1 = vt
= (30m/s)(58) = 150m
=
For the next 3 seconds, find the change in velocity and use it to calculate the
distance:
Av = at = (-2m/s2)(3s) = −6m/s
·
d2 = | | (vƒ + vi)t = }(−6m/s + 30m/s)(38) = 36m
·
The total distance is the sum of d1 and d2:
dtotal
= 150m + 36m = 186m
α=
58
Δυ
-20m/s
=
58
-4m/s2
10. Problem: An object moves with an initial velocity of 5 m/s, and after 10
seconds, its
final velocity is 25 m/s. Calculate the acceleration.
Δυ
Solution: Use the equation: a = 4o
25m/s-5m/s
10s
t
20m/s = 2m/s2
10s
GILZA, WILSON
Problem 1:
From rest, a car accelerated at 8 m/s2 for 10 seconds.
a) What is the position of the car at the end of the 10 seconds?
b) What is the velocity of the car at the end of the 10 seconds?
Solution to Problem 1:
a) The car starts from rest therefore the initial speed u = 0. Nothing is said about
the initial position and we therefore assume it is equal to 0. Hence the position x
is given by the equation
x = (1/2) a t 2
where a is the acceleration (=8 m/s2) and t is the period of time between initial
and final positions
x = (1/2)8 (10)2 = 400 m
b) The velocity v of the car at the end of the 10 seconds is given by
v = a t = 8 * 10 = 80 m/s
Problem 2:
With an initial velocity of 20 km/h, a car accelerated at 8 m/s2 for 10 seconds.
a) What is the position of the car at the end of the 10 seconds?
b) What is the velocity of the car at the end of the 10 seconds?
Solution to Problem 2:
a) The car has an initial velocity of 20 km/h, therefore the initial speed u = 20
km/h. Nothing is said about the initial position and we therefore assume it is
equal to 0. Hence the position x is given by the equation
x = (1/2) a t 2 + u t
where a is the acceleration (=8 m/s2) and t is period of time between initial and
final positions and u is the initial velocity.
Since the time is given in seconds, we need to convert 20 km/h into m/s as
follows:
u = 20 km/h =
20 * 1km
1 hour
1000 m
1 km
1 hour
3600 seconds
= 5.6 m/s
We now have
x = (1/2) (8) 10 2 + 5.6*10 = 456 m
b) v = at + u = 8*10 + 5.6 = 85.6 m/s
Problem 3:
A car accelerates uniformly from 0 to 72 km/h in 11.5 seconds.
a) What is the acceleration of the car in m/s2?
b) What is the position of the car by the time it reaches the velocity of 72 km/h?
Solution to Problem 3:
a) The acceleration a is a measure if the rate of change of the velocity within a
period of time. Hence
u=
change in velocity
change in time
=
v-u
t
=
72 km/h - 0
11.5 seconds
We now convert 72 km/h into m/s
u = 72 km/h =
72 * 1km
1 hour
1000 m
1 km
1 hour
3600 seconds
= 20 m/s
We now calculate the acceleration a
a = (20 m/s) / (11.5 s) = 1.74 m/s 2 (approximetd)
b) Two ways to find the position x:
1) x = (1/2)(v + u) t or 2) x = (1/2) a t 2 + u t
1) We first use: x = (1/2)(v + u) t = 0.5*(20 m/s + 0)*11.5 = 115 m
2) We now use: (1/2) a t 2 + u t = 0.5*1.74*(11.5) 2 + 0*t = 115 m
Problem 4:
An object is thrown straight down from the top of a building at a speed of 20 m/s.
It hits the ground with a speed of 40 m/s.
a) How high is the building?
b) How long was the object in the air?
Solution to Problem 4:
a) We consider that the direction from ground up is the positive direction of the
falling object. We are given the initial (-20 m/s) and final velocities (-40 m/s); the
minus sign was added to take into account the fact that the falling object is
moving in the negative direction. We know the gravitational acceleration (g = -
9.8 m/s2) acting on the falling object and we are asked to find the height of the
building. If we consider the position of the object as being x (wth x = 0 on the
ground), then we may use the equation relating the initial and final velocities u
and v, the acceleration a and the initial (x0 which the height of the building) and
final (x, on the ground) positions as follows:
v2 = u2 + 2 a (x - x0)
(-40 m/s)2 = (-20 m/s)2 + 2 (-9.8 m/s0) (0 - x0)
Solve the above for x0
x0 = 1200 / 19.6 = 61.2 m
b) x - x0 = (1/2)(u + v)t
-61.2 = 0.5(-20 - 40)t
t = 61.2 / 30 = 2.04 s
Problem 5:
A train brakes from 40 m/s to a stop over a distance of 100 m.
a) What is the acceleration of the train?
b) How much time does it take the train to stop?
Solution to Problem 5:
a) We are given the initial velocity u = 40 m/s, the final velocity v = 0 (train stops)
and the distance. Hence the formula that relates these 3 quantities and the
acceleration is given by
v2 = u2 + 2 a x
02 = 402 + 2 a (100)
Solve for the acceleration a
a = -1600 / 200 = - 8 m/s2
b) There two ways to find the time:
1) Use: x = (1/2)(v + u) t
100 = 0.5(0 + 40) t
Solve for t: t = 5 seconds.
2) Use x = (1/2) a t2 + ut
100 = 0.5 ( - 8) t2 + 40t
4 t2 - 40 t + 100 = 0
4 (t2 - 10 t + 25) = 0
4(t - 5)2 = 0
t = 5 seconds.
Problems on velocity and uniform acceleration are presented along with detailed
solutions. Tutorials can also be found in this website.
Problem 1:
From rest, a car accelerated at 8 m/s2 for 10 seconds.
a) What is the position of the car at the end of the 10 seconds?
b) What is the velocity of the car at the end of the 10 seconds?
Solution to Problem 1:
a) The car starts from rest therefore the initial speed u = 0. Nothing is said about
the initial position and we therefore assume it is equal to 0. Hence the position x
is given by the equation
x = (1/2) a t 2
where a is the acceleration (=8 m/s2) and t is the period of time between initial
and final positions
x = (1/2)8 (10)2 = 400 m
b) The velocity v of the car at the end of the 10 seconds is given by
v = a t = 8 * 10 = 80 m/s
Problem 2:
With an initial velocity of 20 km/h, a car accelerated at 8 m/s2 for 10 seconds.
a) What is the position of the car at the end of the 10 seconds?
b) What is the velocity of the car at the end of the 10 seconds?
Solution to Problem 2:
a) The car has an initial velocity of 20 km/h, therefore the initial speed u = 20
km/h. Nothing is said about the initial position and we therefore assume it is
equal to 0. Hence the position x is given by the equation
x = (1/2) a t 2 + u t
where a is the acceleration (=8 m/s2) and t is period of time between initial and
final positions and u is the initial velocity.
Since the time is given in seconds, we need to convert 20 km/h into m/s as
follows:
u = 20 km/h =
20 * 1km
1 hour
1000 m
1 km
1 hour
3600 seconds
= 5.6 m/s
We now have
x = (1/2) (8) 10 2 + 5.6*10 = 456 m
b) v = at + u = 8*10 + 5.6 = 85.6 m/s
Problem 3:
A car accelerates uniformly from 0 to 72 km/h in 11.5 seconds.
a) What is the acceleration of the car in m/s2?
b) What is the position of the car by the time it reaches the velocity of 72 km/h?
Solution to Problem 3:
a) The acceleration a is a measure if the rate of change of the velocity within a
period of time. Hence
u=
change in velocity
change in time
=
v-u
t
=
72 km/h - 0
11.5 seconds
We now convert 72 km/h into m/s
u = 72 km/h =
72 * 1km
1 hour
1000 m
1 km
1 hour
3600 seconds
= 20 m/s
We now calculate the acceleration a
a = (20 m/s) / (11.5 s) = 1.74 m/s 2 (approximetd)
b) Two ways to find the position x:
1) x = (1/2)(v + u) t or 2) x = (1/2) a t 2 + u t
1) We first use: x = (1/2)(v + u) t = 0.5*(20 m/s + 0)*11.5 = 115 m
2) We now use: (1/2) a t 2 + u t = 0.5*1.74*(11.5) 2 + 0*t = 115 m
Problem 4:
An object is thrown straight down from the top of a building at a speed of 20 m/s.
It hits the ground with a speed of 40 m/s.
a) How high is the building?
b) How long was the object in the air?
Solution to Problem 4:
a) We consider that the direction from ground up is the positive direction of the
falling object. We are given the initial (-20 m/s) and final velocities (-40 m/s); the
minus sign was added to take into account the fact that the falling object is
moving in the negative direction. We know the gravitational acceleration (g = -
9.8 m/s2) acting on the falling object and we are asked to find the height of the
building. If we consider the position of the object as being x (wth x = 0 on the
ground), then we may use the equation relating the initial and final velocities u
and v, the acceleration a and the initial (x0 which the height of the building) and
final (x, on the ground) positions as follows:
v2 = u2 + 2 a (x - x0)
(-40 m/s)2 = (-20 m/s)2 + 2 (-9.8 m/s0) (0 - x0)
Solve the above for x0
x0 = 1200 / 19.6 = 61.2 m
b) x - x0 = (1/2)(u + v)t
-61.2 = 0.5(-20 - 40)t
t = 61.2 / 30 = 2.04 s
Problem 5:
A train brakes from 40 m/s to a stop over a distance of 100 m.
a) What is the acceleration of the train?
b) How much time does it take the train to stop?
Solution to Problem 5:
a) We are given the initial velocity u = 40 m/s, the final velocity v = 0 (train stops)
and the distance. Hence the formula that relates these 3 quantities and the
acceleration is given by
v2 = u2 + 2 a x
02 = 402 + 2 a (100)
Solve for the acceleration a
a = -1600 / 200 = - 8 m/s2
b) There two ways to find the time:
1) Use: x = (1/2)(v + u) t
100 = 0.5(0 + 40) t
Solve for t: t = 5 seconds.
2) Use x = (1/2) a t2 + ut
100 = 0.5 ( - 8) t2 + 40t
4 t2 - 40 t + 100 = 0
4 (t2 - 10 t + 25) = 0
4(t - 5)2 = 0
t = 5 seconds.
Problem 6:
A boy on a bicycle increases his velocity from 5 m/s to 20 m/s in 10 seconds.
a) What is the acceleration of the bicycle?
b) What distance was covered by the bicycle during the 10 seconds?
Solution to Problem 6:
a) In this problem the initial velocity u = 5 m/s and the final velocity v = 20 m/s.
The acceleration a of the bicycle is the rate of change of the velocity and is given
as follows
a=
v-u
t
=
20 m/s - 5 m/s
10 seconds
= 1.5 m/s2
b) There are two ways to find the distance covered by the bicyle in t = 10
seconds.
1) x = (1/2)(v + u) t = 0.5 (20 + 5) 10 = 125 m
2) x = (1/2) a t 2 + u t = 0.5 * 1.5 * 100 + 5 * 10 = 125 m
Problem 7:
a) How long does it take an airplane to take off if it needs to reach a speed on the
ground of 350 km/h over a distance of 600 meters (assume the plane starts from
rest)?
b) What is the acceleration of the airplane over the 600 meters?
Solution to Problem 7:
a) In this problem the initial velocity u = 0 (assumed because it is not given) , the
final velocity v = 350 km/h and the distance x = 600 meters = 0.6 km
The relationship between the give quantities is:
x = (1/2)(v + u) t
0.6 = 0.5 (350 + 0) t
Solve for t
t = (0.6 / 175) hours = 12.3 seconds
b) The acceleration a of the airplane is given by
a = (v - u) / t = 350 km/h / 12.3 s
Convert 350 km/h into m/s
350 km/h = 350,000 m / 3,600 s = 97.2 m/s
a = 97.2 m/s / 12.3 s = 8 m/s2 (to the nearest unit)
Problem 8:
Starting from a distance of 20 meters to the left of the origin and at a velocity of
10 m/s, an object accelerates to the right of the origin for 5 seconds at 4 m/s2.
What is the position of the object at the end of the 5 seconds of acceleration?
Solution to Problem 8:
a) In this problem, we may consider that the direction of the object is the positive
direction and the initial position x0 = -20 meters (to the left of the origin), the initial
velocity u = 10 m/s, the acceleration a = 4 m/s2 and the time is t = 5 seconds.
The position is given by
x = (1/2) a t2 + u t + x0
= 0.5 * 4 * (5)2 + 10 * 5 - 20 = 80 meters to the right of the origin.
Problem 9:
What is the smallest distance, in meters, needed for an airplane touching the
runway with a velocity of 360 km/h and an acceleration of -10 m/s2 to come to
rest?
Solution to Problem 9:
a) In this problem the initial velocity u = 360 km/h, the final velocity v = 0 (rest)
and the acceleration a = -10 m/s2. The distance x can be calculated using the
formula
v2 = u2 + 2 a x
Convert 360 km/h into m/s: 360 km/h = (360 000 m) /(3600 s) = 100 m/s
x = ( v2 - u2 ) / (2 a) = (0 - 10,000) / (-20) = 500 meters
Problem 10:
To approximate the height of a water well, Martha and John drop a heavy rock
into the well. 8 seconds after the rock is dropped, they hear a splash caused by
the impact of the rock on the water. What is the height of the well. (Speed of
sound in air is 340 m/s).
Solution to Problem 10:
a) In this problem we have:
1) a rock was dropped down the well and is uniformly accelerated downward due
to gravity. If h is the height of the well and t is the time taken by the rock to reach
the bottom of the well, then we have
h = (1/2)(9.8) t 2
2) After the splash, the sound travels up the well at a constant speed of 340 m/s.
Again the same height h of the well is given by
h = 340 *(8 - t) : 8 - t is the time taken for the sound to travel from bottom to top
where the sound is heard.
The above equations give:
(1/2)(9.8) t2 = 340 *(8 - t)
4.9 t2 + 340 t - 2720 = 0
Solve for t, two solutions:
t = 7.24 s and the second solution is negative and is not valid.
The height h of the well is calculated using one of the above equations:
h = 340 *(8 - t) = 340 *(8 - 7.24) = 257 meters (approximated to the the nearest
meter)
GONDRA, TYRONE NATHANIEL
1. Problem: A spaceship is traveling at 0.8 times the speed of light (c). Calculate
its relativistic mass. (Use the relativistic mass formula: m = m0 / √(1 - v²/c²),
where m0 is the rest mass, and v is the velocity)
6. Problem: A prism disperses white light into its constituent colors. If the red light
has a wavelength of 700 nm and the violet light has a wavelength of 400 nm,
calculate the refractive index of the material.
10. Problem: An object is in free fall from a height of 50 meters. Calculate its final
velocity just before hitting the ground.
acceleration?
Solution: We can use the equation of motion: v = u + at, where v is the final
velocity, u is the initial velocity (0 m/s in this case), a is the acceleration, and t is
the time. Rearranging the equation, we get a = (v - u)/t = (30 m/s - 0 m/s)/10 s =
3 m/s².
9. Problem: A bullet is fired with a velocity of 400 m/s and takes 0.05 seconds to
travel through a wooden block. What is the thickness of the block?
Solution: We can use the formula distance = velocity * time. Plugging in the
values, we have distance = 400 m/s * 0.05 s = 20 meters.
10. Problem: An object is moving in a straight line with an initial velocity of 10 m/s
and a constant acceleration of 2 m/s². What is its final velocity after 5 seconds?
Solution: We can use the equation of motion: v = u + at, where v is the final
velocity, u is the initial velocity, a is the acceleration, and t is the time. Plugging in
the values, we get v = 10 m/s + 2 m/s² * 5 s = 20 m/s.
IGNACIO, ARTJOHN
PROBLEM SOLVING
DISPLACEMENT
Given:
11.2 mg to grams
Unit Conversion:
mg to grams
Solution:
11.2mg x 1g
1000mg
= 0.0112g
Given:
9,800,000m to miles
Unit conversion:
mm to miles
Solution:
=6.089426 miles
3. A car travels 23km in 15 minutes. How fast is it going in kilometers per hour?
23km
15 minutes
Solution:
23km x 60mins
15mins 1h = 92km/h
92km x 1000m x 1h
1h 1km 3600s = 25.6 m/s
4. Blood sugar level are measured in milligrams of glucose per decilitre of blood
Given:
128mg/dL to g/L
Solution:
7,200,000 x 5,000,000
Solution:
Solution:
= (5.7 x 104) + (4.87 x 105)
= (0.57 x 105) + (4.87 x 105)
= (0.57 + 4.87) x 105
= 5.44 x 105
Formula: c2 = a2 + b2
Given:
F1= 5.0m, E
F2= 10.0m, N
Fr=?
Solution:
c2 = √a2 + b2
= √(5.0m, E)2 + (10.0m, N)2
= √25m, E2 + 10.0m, N2
= √125m, NE
Fr = 11.1m, NE
8. Car's speedometer reads 10,500km at the start of a trip and 10,700km at the
Solution:
9. A runner travels around rectangle track with lenght of 50m and width of 20m.
After travels around rectangle rwck for 2 times, the runner goes back to the
Given:
length = 50m
width = 20m
Solution:
circumference of rectangle = 2(50m) + 2(20m)
= 100m + 40m
= 140m
10. A student is driving their car to school. They start by driving 8.33km north,
then turn eastward to travel the final 3.67km to the school. What is the
Given:
F1 = 8.33km, North
F2 = 3.67km, East
Fr = ?
Solution:
Formula: c2 = √a2 + b2
c2 = √a2 + b2
= √(8.33km, N)2 + (3.67km, E)2
= √69.4km,N2 + 13.5km, E2
= √82.9km, NE
= 9.10km, NE
JULIANE, JELTHRUDE
Q1.A boat is moving with velocity of 5 m/s in the x direction and 2 m/s in they
direction. Find resulting velocity and the angle of the boat's motion.
A = 30.0 N at 0°
B = 40.0 N at 90
Q3.headwind and crosswind, total wind velocity of 15 m s at angle 30° from the
landing direction.
Q4.It takes a force of 25 pounds to keep a box from sliding down a ramp that is
inclined at 21º from the horizontal. How much does the box weigh?
Q5.An airplane accelerates down a runway at 3.20 m/s2 for 32.8 s until is finally
lifts off the ground. Determine the distance traveled before takeoff.
Given: Find:
a = +3.2 m/s t = 32.8 s vi = 0 m/s d = ??
2
d = vi*t + 0.5*a*t2
d = (0 m/s)*(32.8 s)+ 0.5*(3.20 m/s2)*(32.8 s)2
d = 1720 m
Q6.A car starts from rest and accelerates uniformly over a time of 5.21 seconds
for a distance of 110 m. Determine the acceleration of the car.
Given: Find:
d = 110 t = 5.21 s vi = 0 m/s a = ??
m
d = vi*t + 0.5*a*t2
110 m = (0 m/s)*(5.21 s)+ 0.5*(a)*(5.21 s)2
110 m = (13.57 s2)*a
a = (110 m)/(13.57 s2)
a = 8.10 m/ s2
Q7.Upton Chuck is riding the Giant Drop at Great America. If Upton free falls for
2.60 seconds, what will be his final velocity and how far will he fall?
Given: Find:
a = -9.8 m t = 2.6 vi = 0 m/s d = ??
s vf = ??
2
d = vi*t + 0.5*a*t
d = (0 m/s)*(2.60 s)+ 0.5*(-9.8 m/s2)*(2.60 s)2
d = -33.1 m (- indicates direction)
vf = vi + a*t
vf = 0 + (-9.8 m/s2)*(2.60 s)
vf = -25.5 m/s (- indicates direction)
Q8.A race car accelerates uniformly from 18.5 m/s to 46.1 m/s in 2.47 seconds.
Determine the acceleration of the car and the distance traveled.
Given: Find:
vi = 18.5 m/s vf = 46.1 m/s t = 2.47 s d = ??
a = ??
a = (Delta v)/t
a = (46.1 m/s - 18.5 m/s)/(2.47 s)
a = 11.2 m/s2
d = vi*t + 0.5*a*t2
d = (18.5 m/s)*(2.47 s)+ 0.5*(11.2 m/s2)*(2.47 s)2
d = 45.7 m + 34.1 m
d = 79.8 m
Q9.A feather is dropped on the moon from a height of 1.40 meters. The
acceleration of gravity on the moon is 1.67 m/s2. Determine the time for the
feather to fall to the surface of the moon.
Given: Find:
vi = 0 m/s d = -1.40 m a = -1.67 m/s2 t = ??
d = vi*t + 0.5*a*t2
-1.40 m = (0 m/s)*(t)+ 0.5*(-1.67 m/s2)*(t)2
-1.40 m = 0+ (-0.835 m/s2)*(t)2
(-1.40 m)/(-0.835 m/s2) = t2
1.68 s2 = t2
t = 1.29 s
Given: Find:
vi = 0 m/s vf = 444 m/s t = 1.83 s a = ??
d = ??
a = (Delta v)/t
a = (444 m/s - 0 m/s)/(1.83 s)
a = 243 m/s2
d = vi*t + 0.5*a*t2
d = (0 m/s)*(1.83 s)+ 0.5*(243 m/s2)*(1.83 s)2
d = 0 m + 406 m
d = 406 m
LAGRADA, JOSEPH ANGELO
Then we can calculate the final velocity (v) using the formula v = u + at, where u
is the initial velocity (which is zero because the object is at rest), a is
acceleration, and t is time:
v = 0 + (4 m/s² * 3 s)
v = 12 m/s
Question B:
Given:
Mass (m) = 5 kg
Force (F) = 40 N
Time (t) = 3 s
Question C:
Given:
Mass (m) = 5 kg
Force (F) = 20 N
Time (t) = ?
Acceleration (a) = ?
Then we can calculate the time (t) with the formula t = v – u / a, where u is the
initial velocity (which is zero because the object is at rest), a is acceleration, and
v is final velocity:
t = 20 m/s – 0 / 4 m/s²
t=5s
So, the time it would take the object is 5 s.
Then we can calculate the force (F) using the formula F = ma:
F = 1500 kg * -2 m/s²
F = -3000 N
The negative sign indicates that the force is applied in the opposite direction to
the motion of the car. So, the force applied by the brakes is 3000 N.
To find out how far the car travels before it comes to a stop, we can use the
equation of motion: s = ut + 0.5at², where s is the distance traveled:
s = (20 m/s * 10 s) + 0.5 * (-2 m/s²) * (10 s)²
s = 200 m - 100 m
s = 100 m
So, the car travels 100 meters before it comes to a stop.
Question B:
Given:
Then we can calculate the force (F) using the formula F = ma:
F = 1500 kg * -2 m/s²
F = -3000 N
So, if the car was moving at a speed of 30 m/s and came to a stop in 15
seconds, the force applied by the brakes would still be 3000 N.
To find out how far this car travels before it comes to a stop:
So, this car would travel 225 meters before it comes to a stop.
Question C:
Given:
To find out how far the car travels before it comes to a stop, we can use the
equation of motion: s = ut + 0.5at², where s is the distance traveled:
So, it would take approximately 15 seconds for the car to come to a stop and it
would travel about 300 meters in this time.
The time (t) it takes for the biker to reach a final velocity of 3 m/s in the leftward
direction can be calculated using the formula t = (v - u) / a:
t = (3 m/s - 0) / 1.5 m/s²
t = 2 seconds
To find out how far the biker travels in the leftward direction during this time, we
can use the equation of motion: s = ut + 0.5at², where s is the distance traveled:
So, it would take 2 seconds for the biker to reach a final velocity of 3 m/s in the
leftward direction and they would travel 3 meters in this time.
Question A:
Given:
So, if the rocket’s engines are supplying an upward force of 75050 N, the
acceleration of the rocket would be 75.05 m/s².
Question C:
Given:
Problem 5:
A 1 kg apple falls from a tree branch located 10 meters above the ground. How
much kinetic energy will the apple have right before it hits the ground? Ignore air
resistance.
Solution:
The apple’s potential energy at the start is converted into kinetic energy as it
falls. The potential energy (PE) is given by the formula:
PE = m ⋅ g ⋅ h
where:
m is the mass of the object,
g is the acceleration due to gravity, and
h is the height from which the object falls.
Problem 6:
A 1500W electric heater is used to heat a room for 2 hours. How much energy in
joules does the heater use?
Solution:
The energy (E) used by an electric device is given by the formula:
E=P⋅t
where:
P is the power of the device, and
t is the time for which the device is used.
First, we need to convert the power to joules per second (1W = 1J/s):
P = 1500 W
P = 1500 J/s
Then we need to convert the time to seconds (1 hour = 3600 seconds):
T = 2 hours
T = 7200 s
Then we substitute these values into the formula:
E = 1500 J/s ⋅ 7200 s
E = 10800000 J
So, the heater uses 10,800,000 Joules of energy.
Problem 7:
You have a 500g block of ice at 0°C and you want to completely melt it. How
much energy in joules do you need?
Solution:
The energy (Q) needed to melt a substance is given by the formula:
Q = m ⋅ L_f
where:
m is the mass of the substance, and
L_f is the latent heat of fusion.
Problem 8:
You have a 1 kg pot of water at 100°C and you want to completely boil it. How
much energy in joules do you need?
Solution:
The energy (Q) needed to boil a substance is given by the formula:
Q = m ⋅ L_v
where:
m is the mass of the substance, and
L_v is the latent heat of vaporization.
The latent heat of vaporization of water is 2260 J/g. But since we have the mass
in kg, we need to convert it to grams (1 kg = 1000 g).
Substituting the given values:
Q = 1000 g ⋅ 2260 J/g
Q = 2260000 J
So, you need 2,260,000 Joules of energy to completely boil the 1 kg pot of water.
Problem 9:
You have a 200g block of aluminum at 20°C and you want to heat it to 100°C.
How much energy in joules do you need?
Solution:
The energy (Q) needed to heat a substance is given by the formula:
Q = m ⋅ c ⋅ ΔT
where:
m is the mass of the substance,
c is the specific heat capacity of the substance, and
ΔT is the change in temperature.
Q = m ⋅ c ⋅ ΔT
Q = 1000 g ⋅ 4.18 J/g°C ⋅ (25°C−20°C)
Q = 20900 J
Answer: The force of friction acting on the block is given by the formula Ffriction
= μN, where μ is the coefficient of friction and N is the normal force (which is
equal to the weight of the block,since it is on a horizontal surface). Therefore,
Ffriction = 0.3 x (5 kg x 9.8 m/s²) = 14.7 N. The net force on the block is therefore
Fnet = 20 N - 14.7 N = 5.3 N. Using the formula Fnet = ma, we can find the
acceleration of the block: a = Fnet/m = 5.3 N / 5 kg = 1.06 m/s².
2) A ball is thrown horizontally off a cliff with a velocity of 20 m/s. If the cliff is 100
m high, how far from the base of the cliff will the ball land?
Answer: Since the ball is thrown horizontally, its initial vertical velocity is zero.
The only force acting on the ball is gravity, which causes it to accelerate
downwards with a constant acceleration of 9.8 m/s². We can use the formula d =
vit + 1/2at² to find the vertical distance the ball falls while it travels horizontally.
Since vi = 0, this simplifies to d = 1/2at² = 1/2 x 9.8 m/s² x
(10 s)² = 490 m. Therefore, the ball will land 490 m horizontally from the base of
the cliff.
Answer: The force required to keep the object from sliding down the plane is
equal and opposite to the force of gravity that is acting to pull the object down the
plane. The force of gravity is given by Fg = mg, where m is the mass of the object
and g is the acceleration due to gravity (9.8
m/s²). The force that is acting perpendicular to the plane (the normal force) is
equal to the component of the force of gravity that is perpendicular to the plane,
which is given by Fn = mgsinθ, where θ is the angle of the plane with respect to
the horizontal. Therefore, the force required to keep the object from sliding down
the plane is F = mgcosθ. Plugging in the values, we get F = 1 kg × 9.8 m/s² × cos
30° = 8.5 N.
Answer: The force that keeps the ball moving in a circle is the tension in the
string, which acts towards the center of the circle and provides the centripetal
force. The formula for centripetal force is F = mv²/r, where m is the mass of the
ball, v is its speed (which is equal to the circumference of the circle divided by the
time it takes to make one revolution), and r is the radius of the circle. Therefore, v
= 2πr/t = 2π × 1 m / 2 s = π m/s. Plugging in the values, we get F = 2 kg × (π
m/s)² / 1 m = 9.87 N.
6) A bullet with a mass of 0.05 kg is fired from a rifle with a velocity of 400 m/s. If
the bullet is brought to rest by a target with a mass of 10 kg, what is the velocity
of the target?
Since momentum is conserved, we can equate the initial and final momenta: p1 =
p2, which gives 20 kg m/s = 0 kg m/s + 10 kg × v2. Solving for v2, we get v2 =
20/10 = 2 m/s.
Answer: The power of the motor is the rate at which it does work, which is equal
to the force it applies times the distance it moves the load in a given time. The
work done is given by W = Fd, where F is the force exerted by the motor and d is
the distance the load is lifted. Therefore, the power of the motor is P = W/t = Fd/t.
Plugging in the values, we get P = (2000 N) × (20 m) / 30 s = 1333 W.
10) A car of mass 1000 kg traveling at a speed of 20 m/s collides head-on with
another car of mass 1500 kg traveling at a speed of 15 m/s in the opposite
direction. After the collision, the two cars stick together and move as a single
unit. What is their final velocity?
2.a car accelerates from rest to 100km/h in 20 seconds what is the acceleration
Vf = vi + a t
vf = final velocity
vi = initial velocity
a = acceleration
t = elapsed time
100km/h = 0.005556 h. a
Therefor the acceleration is 1.7999.km/h
4.a train travel 200 km in 5 hours. What is the average speed of the train?
V = average velocity
X = Displacement
T = Change in time
x = 200 km t = 5 h = 40km/h
Therefoe the average speed of train is 40km/h
Kinematics
1. A car accelerates uniformly from rest to 60 mph in 5 seconds. What is the car’s
acceleration?
**Solution:**
We can use the following equation to solve for the car’s acceleration:
A = (v_f – v_i) / t
Where:
* a is the acceleration
* v_f is the final velocity
* v_i is the initial velocity
* t is the time
2. A ball is thrown vertically upward with an initial velocity of 20 m/s. How high
does
the ball reach?
**Solution:**
We can use the following equation to solve for the ball’s maximum height:
H = v_i^2 / 2g
Where:
* h is the maximum height
* v_i is the initial velocity
* g is the acceleration due to gravity
A = 20 N / 10 kg = 2 m/s^2
Therefore, the block’s acceleration is 2 m/s^2.
W = 98.1 N * 20 m = 1962 J
Therefore, 1962 J of work is done.
At the bottom of the ramp, the box’s potential energy has been converted into
kinetic
energy. Therefore, the box’s kinetic energy is equal to the work done on the box
by gravity.
The work done by gravity is equal to the force of gravity multiplied by the
distance the box
slides:
W = F_g * d = mg * sin(theta) * d
KE = 1⁄2 * mv^2 = mg * sin(theta) * d
Circular Motion
7. Problem: A car is traveling around a circular track with a radius of 50 m at a
speed of
20 m/s. What is the centripetal acceleration of the car?
Solution:
Centripetal acceleration:
Ac = v^2 / r = 20 m/s^2 / 50 m = 8.0 m/s^2
yo-yo is 0.20 m. What is the centripetal acceleration of a point on the edge of the
yo-
yo?
Solution:
Centripetal acceleration:
Ac = v^2 / r = (2 * pi * 10 rev/s * 0.20 m)^2 / 0.20 m = 400 pi^2 m/s^2
Dynamics
9. Problem:Rocket-powered sleds are used to test the human response to
acceleration. If a rocket-powered sled is accelerated to a speed of 444 m/s in
1.83
seconds, then what is the acceleration and what is the distance that the sled
travels?
Solution:
Acceleration:
A = 444 m/s / 1.83 s = 243 m/s^2
Distance traveled:
D = 444 m/s * 1.83 s = 809 m
3. A piece of silicon wafer measures 10.2 mm x 6.5mm x2.8 mm. Following the
correct number of significant figures, what is the volume of the silicon wafer?
Ans: 190 mm3
7. A 12kg block slides down a 24o incline. If the coefficient of friction between the
block and the incline is 0.2, determine the acceleration of the block as it slides
down the incline
Ans: 2.2ms2
8. If a 15kg object is subjected to a force of 175N to the east and another force of
1020N to the west, what is the object’s acceleration?
Ans: 56m/s2
10. A boy kicks a box, giving it a speed of 6.0 m/s. The box slows down to a stop
after travelling a distance of 3.0 m. What is the acceleration of the box?
Ans: 6.0 m/s2
MONTE, RALPH LAUREN
1. A car travels at a constant speed of 60 km/h. How far will it travel in 2 hours?
Solution: Distance = Speed × Time = 60 km/h × 2 h = 120 km
2. An object falls freely from a height of 45 meters. What is its final velocity just
before hitting the ground?
Solution: Final Velocity = √(2 × acceleration × height) = √(2 × 9.8 m/s² × 45 m)
≈ 33.4 m/s
3. A ball is thrown vertically upwards with an initial velocity of 20 m/s. How high
will it go?
Solution: Maximum height = (Initial Velocity²) / (2 × acceleration due to gravity)
= (20 m/s)² / (2 × 9.8 m/s²) ≈ 20.4 m
5. A bullet is fired horizontally with a velocity of 500 m/s. How far will it travel in 3
seconds?
Solution: Distance = Velocity × Time = 500 m/s × 3 s = 1500 m
7. A pendulum swings back and forth with a period of 2 seconds. What is its
frequency?
Solution: Frequency = 1 / Period = 1 / 2 s = 0.5 Hz
8. A rocket accelerates at a rate of 25 m/s². How fast will it be traveling after 10
seconds?
Solution: Final Velocity = Initial Velocity + (Acceleration × Time) = 0 m/s + (25
m/s² × 10 s) = 250 m/s
10. A marble rolls down a ramp with an acceleration of 4 m/s². What is its initial
velocity?
Solution: Initial Velocity = √(Final Velocity² - 2 × acceleration × distance) = √(0²
- 2 × 4 m/s² × 5 m) ≈ -4 m/s
MOYA, KARTHCAREVIN
Answer: 1 cm/s2.
2.A train of 150 meters long is going towards north direction at a speed of 10
m/s. A parrot flies at the speed of 5 m/s towards south direction parallel to the
railway track. The time taken by the parrot to cross the train is
a. 12 sec.
b. 8 sec
c. 15 sec
d. 10 sec
Answer: D.
4. The ratio of the dimension of Planck’s constant and that of the moment of
inertia is the dimension of
a.time
b. frequency
c. angular momentum
d. velocity
Answer: B.
Answer: C.
7. In a particular system, the unit of length, mass and time are chosen to be 10
cm, 10 g and 0.1 s respectively. The unit of force in this system will be equivalent
to
a 0.1 N
b. 1 N
c.10 N
d. 100 N
Answer: A.
8. The density of a cube is measured by measuring its mass and length of its
sides. If the maximum error in the measurement of mass and length are 4% and
3% respectively, the maximum error in the measurement of density will
a. 7%
b. 9%
c.12%
d. 13%
Answer: D.
10.What is called the frequency of a sound when it is more than 20000 Hz?
Answer.(3rd LawAir will rush out of the balloon forcing the balloon to move
through the air in the opposite direction, but equal in force).
2. Describe what happens if you are riding a skateboard and hit something (like a
curb) with the front wheels.
Answer. (1st LawYour body will keep moving forward and fly off your skateboard
since the curb only stops the board, not yourself.)
3. Describe what happens if you try and push Mr. Larson. What happens if he
pushes back?
Answer. (2nd LawIf you put force on Mr. Larson, force will be put back on you.
Because of Mr. Larson’s mass, you most likely will not have enough force to
make him accelerate. If Mr. Larson pushed you, you most likely would move in
the direction of the force since he has more mass).
4. Describe why you hold your gun next to your shoulder while deer hunting.
Answer. (3rdLawWhen you pull the gun’s trigger, it forces the bullet out of the
gun, but at the same time, the gun is forced in the opposite direction of the bullet
(towards you). Your shoulder is a new force that is introduced in order to keep
your gun from flying away from you).
5. What is another name for the first law of motion? Why is it given that name?
7. A walker goes 1000 m the direction 30 degrees North of East, then 2000 m in
the SouthWest direction. The trip takes 30 minutes.Find the displacement, way
length, average velocity and average speed.
Solution:
The displacement is given by
𝐝 = 𝒓01 + 𝒓12, where 𝒓01
= (𝑟01,𝑥, 𝑟01,𝑦)
= (𝑟01 cos 30°, 𝑟01 sin 30°)
= (1000√32, 100012) = (500√3, 500) m𝒓12
= (𝑟12,𝑥, 𝑟12,𝑦)
= (−𝑟12 cos 45°, −𝑟12 sin 45°)
= (−2000√22, − 2000√22)
= (−1000√2, − 1000√2, )
Better is to write
𝒓12 = (𝑟12,𝑥, 𝑟12,𝑦)
= (𝑟12 cos 125°, 𝑟12 sin 125°)
= (2000 (−√22))2) , 2000 (−√2= (−1000√2, − 1000√2, ) m that gives the same
result.
Now,𝐝 = (𝑟01,𝑥 + 𝑟12,𝑥, 𝑟01,𝑦 + 𝑟12,𝑦)
= (500√3 − 1000√2, 500 − 1000√2)≈ (−548.2, −914.2) m
The distance is given by
𝑑 = |𝐝| = √𝑑𝑥2 + 𝑑𝑦2 = √(−548.2)2 + (−914.2)2 ≈ 1066 m
The length of the trajectory is𝑤 = 𝑟01 + 𝑟12 = 1000 + 2000 = 3000 m.
The velocity:𝐯 =∆𝐫∆𝑡=𝐝∆𝑡=(−548.2, −914.2)30 × 60 = (−548.230 × 60 ,−914.230
× 60) = (… , … ) m/s.
The magnitude of the average velocity:𝑣 = |𝐯| =𝑑∆𝑡=106630 × 60 = 0.59 m/s.
The average speed:∆𝑡=3000𝑠 =𝑤30 × 60 = 1.67 m/s > 𝑣.
8. A car started moving from rest with a constant acceleration. At some moment
of time, it covered the distance 𝑥 and reached the speed 𝑣. Find the acceleration
and the time
Solution. The formulas for the motion with constant acceleration read𝑣 = 𝑎𝑡, 𝑥
=12𝑎𝑡2, where we have taken into account that the motion starts from rest (all
initial values are zero).
If 𝑣 and 𝑥 are given, this is a system of two equations with the unknowns 𝑎 and 𝑡.
This system of equations can be solved in different ways.
First method. For instance, one can express the time from the first equation, 𝑡 =
𝑣/𝑎, and substitute it to the second equation,)2=𝑣2𝑥 =12𝑎 (𝑣2𝑎.
From this single equation for 𝑎 one finds𝑎 =𝑣22𝑥.
Now, one finds the time as𝑣2/(2𝑥)=2𝑥𝑡 =𝑣𝑎=𝑣𝑣.
Second method. Also, one can relate 𝑥 to 𝑣 as follows𝑥 =12𝑎𝑡 × 𝑡 =12𝑣𝑡.
After that one finds𝑡 =2𝑥𝑣,and, further,2𝑥/𝑣=𝑣2𝑎 =𝑣𝑡=𝑣2𝑥.
10. Newton’s laws of motion can be used to calculate the force on a body?
Scientific notation
1. Convert 0.0000768 to scientific notation.
0.0000768 = 7.68 x 10^-5
Measurements
3. John wants to bake a cake and needs 2 cups of sugar. How many grams of
sugar does he need if 1 cup of sugar is equal to 200 grams?
Solution:
1 cup of sugar = 200 grams
2 cups of sugar = 2 x 200 grams
2 cups of sugar = 400 grams
Therefore, John needs 400 grams of sugar.
Solution:
1 meter = 100 centimeters
10 meters = 10 x 100 centimeters
10 meters = 1000 centimeters
Therefore, the length of the room in centimeters is 1000 centimeters.
5. A recipe calls for 2 1/2 tablespoons of oil. How many milliliters of oil is needed
if 1 tablespoon is equal to 15 milliliters?
Solution:
1 tablespoon = 15 milliliters
2 1/2 tablespoons = (2 x 15) + (1/2 x 15)
2 1/2 tablespoons = 30 + 7.5
2 1/2 tablespoons = 37.5 milliliters
Therefore, 37.5 milliliters of oil is needed.
Law of Motion
6. A car of mass 1000 kg is traveling at 50 m/s. Calculate the force required to
bring the car to a complete stop in 20 seconds.
Solution:
u = 50 m/s
v = 0 m/s
t = 20 s
Acceleration, a = (v - u) / t = (0 - 50) / 20 = -2.5 m/s^2 (negative sign suggests
deceleration)
Therefore, the force required to bring the car to a complete stop in 20 seconds is
2500 N.
Solution:
m = 5 kg
u = 0 m/s
F = 20 N
t = 10 s
Using the third law of motion, we can relate force and acceleration to velocity.
v = u + at
v = 0 + 4 m/s^2 x 10 s
v = 40 m/s
Solution:
k = 200 N/m
x = 0.1 m
m = 2 kg
Using Hooke's Law, F = -kx (negative sign indicates the force acts in the opposite
direction to the compression)
Therefore, the acceleration of the object when the spring is released is -10
m/s^2.
Displacement is the change in position of an object from its initial to final position.
Since the car travels along a straight road, its displacement is equal to the
distance it travels, which is 120 miles.
9. A person walks 5 meters to the east, then turns around and walks 8 meters to
the west. What is the total distance traveled, and what is the displacement?
Solution:
Total distance = 5 meters + 8 meters = 13 meters
Displacement is the difference between the initial and final positions, in terms of
distance and direction. In this case, the displacement is:
The negative sign indicates that the person moved to the west, which is the
opposite direction from the initial position.
10. A train travels from station A to station B, which are 100 kilometers apart. The
train first travels 80 kilometers at a speed of 50 km/h and then slows down to 30
km/h for the remaining distance. What is the total time taken by the train to reach
station B?
Therefore, the train takes 2.27 hours to travel from station A to station B.
PASCUAL, MON ARCHIE
numbers
Answer: 6.5X10^8.
3. How many significant figures does the following number have? 470000
Answer: 2
4. How many significant figures does the following number have? 4.57 x 10⁷
Answer: 3
Answer: 184,086
Answer: Inertia
Answer: Motion
Answer: Distance
Answer: Mass
PESTANO, MARK JAMES
Question
1.When you describe an object's speed, you often describe the average over a
time period. Average speed, vavg, is the distance traveled divided by the time
during which the motion occurs?
Answer
vavg=distancetime
You can, of course, rearrange the equation to solve for either distance or time
time = distancevavg.
Suppose, for example, a car travels 150 kilometers in 3.2 hours. Its average
speed for the trip is
A car's speed would likely increase and decrease many times over a 3.2 hour
trip. Its speed at a specific instant in time, however, is its instantaneous speed. A
car's speedometer describes its instantaneous speed.
A drawing is shown of a house on the left and a store on the right. The distance
between the two is labeled three kilometers. A double-arrow vector between the
house and the store is labeled with the equation change in d total equals zero.
Figure 2.8 During a 30-minute round trip to the store, the total distance traveled
is 6 km. The average speed is 12 km/h. The displacement for the round trip is
zero, because there was no net change in position.
WORKED EXAMPLE
Calculating Average Speed
A marble rolls 5.2 m in 1.8 s. What was the marble's average speed?
STRATEGY
We know the distance the marble travels, 5.2 m, and the time interval, 1.8 s. We
can use these values in the average speed equation.
Discussion
Average speed is a scalar, so we do not include direction in the answer. We can
check the reasonableness of the answer by estimating: 5 meters divided by 2
seconds is 2.5 m/s. Since 2.5 m/s is close to 2.9 m/s, the answer is reasonable.
This is about the speed of a brisk walk, so it also makes sense.
Question
2. A pitcher throws a baseball from the pitcher’s mound to home plate in 0.46 s.
The distance is 18.4 m. What was the average speed of the baseball?
Answer
40 m/s
- 40 m/s
0.03 m/s
Question
3.What is an example of a speed problem in physics?
Answer
For example, suppose a car travels a distance of 100 km. The first 50 km it
travels 30 km/h and the second 50 km it travels at 60 km/h. Its average speed
would be distance /(time interval) = (100 km)/[(50 km)/(30 km/h) + (50 km)/(60
km/h)] = 40 km/h.
Question
4.What is speed in physics simple?
Answer
Speed is defined as. The rate of change of position of an object in any direction.
Speed is measured as the ratio of distance to the time in which the distance was
covered. Speed is a scalar quantity as it has only direction and no magnitude.
Question
5.When a runner jogs 12 km north then turns and runs 16 km east in 3 hours?
Answer
A runner jogs 12km north then turns & runs 16km east in 3 hour. (A) We can find
the displacement of the runner from the Pythagorean theorem: d = d x 2 + d y 2 =
( 16 k m ) 2 + ( 12 k m ) 2 = 20 k m .
8. Question: What is the term for the force that opposes the relative motion or
tendency of such motion of two surfaces in contact?
Answer: The force that opposes relative motion is called friction.
9. Question: What is the principle that explains the buoyant force exerted on an
object immersed in a fluid?
Answer: Archimedes' Principle explains the buoyant force and states that it is
equal to the weight of the fluid displaced by the object.
10. Question: What is the law that describes the relationship between the
pressure and volume of a gas at a constant temperature?
Answer: Boyle's Law describes the relationship between the pressure and
volume of a gas at a constant temperature, stating that they are inversely
proportional.
PIOPONGCO, LEMUEL
1. A car travels at a constant speed of 25 m/s for 10 seconds. How far does
it travel?
Answer: The car covers a distance of 250 meters.
2. Problem. A car travels at a constant speed of 30 m/s for 2 hours. How far
does it travel?
Solution:
The speed of the car in meters per seconds is 25 m/s because 1 kilometer
is equal to 1000 meters, so 90 km/h x (1000 m/km) / 3600 s/h = 25 m/s.
10. Problem. The density of an object is 5 grams per cubic centimeter. What is
its density in kilograms per cubic meter?
Solution:
Answer: Coulombs/Second
second. How long Does it take the ball to return to the boy’s hand?
Answer: Constant
9. The force acting between two point charges can be computed using which
space
SENIDO, NEIL ADRIAN
2. The branch that deals with the motion of bodies without concerning its cause is
called?
answer: Kinematics
4. The branch that examines forces and their impacts on motion is called?
answer: Dynamics
5. When a bus suddenly takes a turn, the passengers are thrown outwards
because of?
answer: inertia of direction
8. when to remove the dust particles the mat is hit with the stick or against the
wall?
answer: inertia of rest
9. The ________ of an object depends on the mass of the object and the amount
of force applied.
answer: accelaration
10. This is the displacement per unit time and has a specific direction?
answer: velocity
SISON, NORJIE PHIL
2.According to Newton's first law of motion, a moving object that is not acted on
by unbalanced force will _______.
Answer: remain in motion
6.Determine the accelerations that result when a 12-N net force is applied to a 3-
kg object.
Answer: 4m/s2
8.Suppose that a sled is accelerating at a rate of 2 m/s2. If the net force is tripled
and the mass is doubled, then what is the new acceleration of the sled?
Answer: 3m/s2
Ans: Force
3. The word __________ means the change of speed or we can say that
change of velocity.
Ans: Acceleration
Ans: Mass
Ans; Motion
Ans: Scalar
Ans: Vector
Ans: Distance
Ans: Speed
TABUZO, MARK ADRIAN
30m
A 5Om B
Solution
Distance = 50 + 30 + 50 = 130m
Displacement = 30m to north
DC
A 50m B
A car
A 800m B 300m C
Solution:
Distance = 800m + 300m = 1100m
A 800m B 300m C
3. Newton’s law of motion
Problem: the object is at rest, the net force must be zero. Choose up as positive.
So down is negative.
Given: two upward forces of +12N and +8N
Solution: Fnet = +12N + (+8N) + Fg
1 = 20N + Fg
Fg = -20N (down)
4. Problem: what is the net force acting on a 5kg crate that is accelerating at
3m/s2?
Solution: F=ma
=5kg x 3m/s2
= 15N
5. Problem: Dave is riding his skateboard and pushes off the ground with his foot.
This causes him to accelerate at a rate of 4 m/s2 dave weighs 589N. How was
his push off the ground?
Solution: F = ma
= 60kg x 4m/s2
= 240N
9. Your cat has a mass of 2kg. What is her weight in Newtons on earth?
Solution: Fw = mg
= 2kg x 9.8 m/s2
= 19.6 N
10. What force must act on a 50kg mass to give it an acceleration of 0.30 m/s2?
Solution: F = ma
= 50kg x 0.30m/s2
= 15kg m/s2 / 15N
TIPAY, PAUL CRHRISTIAN
Reasoning: Rolling objects down the inclined plane allow him to study their
motion at a lower acceleration than the free fall. See page 24 of the text.
2. An airplane that flies at 100 km/h with a 10 km/h tailwind travels at 110 km/h
relative to the ground. If it instead flies into a 10 km/h headwind, its ground speed
is : A
A. 90 km/h
b. 100 km/h
c. 110 km/h
d. 120 km/h
a. 10 km/h
b. 20 km/h
c. 30 km/h
d. More than 30 km/h
5.What is the acceleration of a car that maintains a constant velocity of 100 km/hr
for 10 seconds? : A
a. 0
b. 10 km/hr/s
c. Both of these
d. None of these
a. Velocity increases
b. Acceleration increases
c. Both of these
d. None of these
A. 10 m/s
b. 20 m/s
c. 30 m/s
d. 40 m/s
e. Depends on its initial speed.
A. Constant
b. Less and less
c. Grater than the second before.
Reasoning: the distance traveled each second increases due to the fact that the
velocity is increasing.
9 Ten seconds after starting from rest, an object falling freely downward will have
a speed of about: C
a. 10 m/s
b. 50 m/s
c. 100 m/s
d. More than 100 m/s
Reasoning: Speed increases at a rate of 10 m/s (actually 9.8 m/s) every second.
Thus after 10 seconds, the speed is 10 x 10 = 100 m/s.
10. If an object falls with constant acceleration, the velocity of the object must: B
a. Be constant also
b. Continuously change by the same amount each second
c. Continuously change by varying amounts depending on its speed
d. Continuously decrease
e. None of these.
UNIDA, ALL VINCENT
1.A man. Walks 1 km due eart and. Then 1 km due north. What is hi
displacement?
IKM. N For. C² = 92 +62 1km, E Given c² = a²+ b² 1 km,
east 1km, north.
Sol.
C² = √ a²+b² = √(1km, E)² + (1km, N)² = √1 km, E² + 1 km, N^ √2 km, NE = 1.42
km. NE
2. Derrick crawls 4m south and then turns east and crawls 6m. Find the resultant
force?
Given F2 = 6m, E 4m S Fr 6m.E Fr = ?
Formula: C² = a² +b²
Sol.
C² = √ a² +62 Fr: Fi+ F22 Fr² = √(4m. 5)2 + (6m. E)² =√16m.s² + 36m. E =
52m.SE Fr 7.21m, CET
3.Convert 24 kim kilometers to meters per second. Per minute 20 24 km min
24kh 1000 m х 605 24 x 1000 m = 244000m x 60 24000 m 605 Spring Leaf =400
m/v
4. convert 1,234 km to m
Sol.
1.234 km x 1000m = 1.234 x 1000m 1234 m
5. If a space shuttle can travel at 17,000 miles per hour, how many meters per
recond doer it travel?
Sol.
1000m 17000 1.609kpx 1000 х 605 = 2 17000 X 1.609 = 27353 x 1000m
=27353000m 6 = 4558833.333333333 m =7-6x 10° m/s
7.An object falls from a height of 80 meters. Calculate the time it takes to reach
the ground. (Assume \(g = 9.8 m/s²\))
Solution 8:
Use the kinematic equation: \(h = \frac{1}{2}gt^2\)
\(80 m = \frac{1}{2} × 9.8 m/s² × t^2\)
Solve for \(t\): \(t = \sqrt{\frac{2 × 80 m}{9.8 m/s²}} ≈ 4.52 seconds.
9. Question: A 25-gram metal ball is heated 200 °C with 2330 Joules of energy.
What is the specific heat of the metal?
Solution: List the information we know.
M = 25 grams
ΔT = 200 °C
Q = 2330 J
Place these into the specific heat equation.
Q = mcΔT
2330 J = (25 g)c(200 °C)
2330 J = (5000 g°C)c
Divide both sides by 5000 g°C
Specific heat example math step 1
C = 0.466 J/g°C
Answer: The specific heat of the metal is 0.466 J/g°C.
10. Question: A 500 gram cube of lead is heated from 25 °C to 75 °C. How much
energy was required to heat the lead? The specific heat of lead is 0.129 J/g°C.
Solution: First, let’s the variables we know.
M = 500 grams
C = 0.129 J/g°C
ΔT = (Tfinal – Tinitial) = (75 °C – 25 °C) = 50 °C
Plug these values into the specific heat equation from above.
Q = mcΔT
Q = (500 grams)·(0.129 J/g°C)·(50 °C)
Q = 3225 J
Answer: It took 3225 Joules of energy to heat the lead cube from 25 °C to 75 °C.
VALENCIA, MARC ANDREY
1. Electricity
Problem: What is the resistance of a 200-ohm resistor when connected in parallel
with a 300-ohm resistor?
Solution:
The formula for calculating the total resistance in parallel is: 1/R_total = 1/R1 +
1/R2, where R1 is the resistance of the first resistor (200 ohms) and R2 is the
resistance of the second resistor (300 ohms).
1/R_total = 1/200 ohms + 1/300 ohms
1/R_total = (3/600 + 2/600) ohms
1/R_total = 5/600 ohms
R_total = 600/5 ohms = 120 ohms
2. Magnetism
Problem: A wire carrying a current of 4 A is placed in a magnetic field of 0.5 T.
Calculate the force acting on the wire if the angle between the current and
magnetic field is 30 degrees.
Solution:
The formula for the magnetic force on a current-carrying wire in a magnetic field
is: F = BILsin(θ), where B is the magnetic field strength (0.5 T), I is the current (4
A), L is the length of the wire, and θ is the angle between the current and the
magnetic field (30 degrees).
F = (0.5 T) * (4 A) * L * sin(30 degrees)
F = (0.5 T) * (4 A) * L * 0.5
F = 1 TAL
3. Modern Physics
Problem: If a photon has a wavelength of 500 nm (nanometers), what is its
energy?
Solution:
The energy of a photon can be calculated using the formula: E = hf, where E is
the energy, h is Planck's constant (6.626 x 10^-34 J·s), and f is the frequency.
First, find the frequency using the speed of light equation: c = λf, where c is the
speed of light (3 x 10^8 m/s) and λ is the wavelength (500 nm).
f = c / λ = (3 x 10^8 m/s) / (500 x 10^-9 m) = 6 x 10^14 Hz
Now, calculate the energy:
E = (6.626 x 10^-34 J·s) * (6 x 10^14 Hz) = 3.98 x 10^-19 J
5. Circular Motion
Problem: A car is moving around a circular track with a radius of 100 meters at a
constant speed of 20 m/s. What is the magnitude of the car's centripetal
acceleration?
Solution:
Centripetal acceleration is given by the formula: a = v² / r, where v is the velocity
(20 m/s) and r is the radius (100 meters).
a = (20 m/s)² / 100 m = 4 m/s²
6. Thermodynamics
Problem: A gas initially at 300 K expands isothermally and does 2000 J of work
on its surroundings. What is the change in its internal energy?
Solution:
For an isothermal process, the change in internal energy is equal to the work
done. So, ΔU = -2000 J.
7. Optics
Problem: A converging lens has a focal length of 15 cm. What is the image
distance for an object placed 30 cm in front of the lens?
Solution:
Use the lens formula: 1/f = 1/v - 1/u, where f is the focal length (15 cm), v is the
image distance (unknown), and u is the object distance (-30 cm because the
object is in front of the lens).
1/15 cm = 1/v - 1/(-30 cm)
1/15 cm = 1/v + 1/30 cm
Multiply both sides by 30v:
2v = 30
v = 15 cm
8. Kinematics
Problem: A car accelerates from rest at a rate of 3 m/s². How long will it take to
reach a speed of 30 m/s?
Solution:
Using the equation: v = u + at, where v is the final velocity (30 m/s), u is the initial
velocity (0 m/s), a is the acceleration (3 m/s²), and t is the time.
30 m/s = 0 m/s + 3 m/s² * t
t = 30 m/s / 3 m/s² = 10 seconds
9. Newton's Laws
Problem: If an object has a mass of 5 kg and is subjected to a force of 20 N, what
is its acceleration?
Solution:
Using Newton's second law: F = ma, where F is the force (20 N), m is the mass
(5 kg), and a is the acceleration.
a = F / m = 20 N / 5 kg = 4 m/s²
Now, use the horizontal motion equation: d = vt, where d is the horizontal
distance, v is the horizontal velocity (15 m/s), and t is the time.
d = 15 m/s * t = 15 m/s * 0 s = 0 meters
VILLANUEVA, JOHN PATRICK
For example, 100000000 can be written as 108, which is the scientific notation.
Here the exponent is positive. Similarly, 0.0000001 is a very small number which
can be representedas10-8, where the exponent is negative
Exponent = 7
Since the exponent is positive we need to move the decimal place 7 places to the
right.
Therefore,
1.36 × 107 = 1.36 × 10000000 = 1,36,00,000
Significant figures are used to demonstrate the number which is presented in the
formof digits. These digits represent numbers in a meaningful way. Instead of
figures, the phrase significant digits is frequently used. By counting all the values
starting with the first non-zero digit ontheleft, we may determine the number of
significant digits. The number 13.75, for example, includes four significant digits.
Any non-zero numbers or trapped zeros are significant figures. Leading and
trailing zerosarenot included.
Q3: Give examples of Significant Figures.
Answer:
Significant Figures examples are as follows:
4308 – 4 significant figures
40.05 – 4 significant figures
470,000 – 2 significant figures
4.00 – 3 significant figures
0.00500 – 3 significant figures
Answer: This measurement has three significant numbers since all nonzero
digits are significant.
The first three zeros are insignificant, but the zero between the sixes is, hence
this number has four significant figures.
This measurement includes four significant figures because the two zeros
between the two are significant.
The four trailing zeros in the number aren’t significant, but the other five are,
making this a five- figure number.
Answer:
Because the first non-significant digit is 5, and we round the last significant figure
up to6tomake it even, 4.7475 becomes 4.748.
Because the first non-significant figure is 5, and the last significant figure is even,
4.7465becomes 4.746.
Q6: A ladder is kept at a distance of 15 cm from the wall such that the top of the
ladder isat theheight of 8 cm from the bottom of the wall. Find the length of the
wall.
Solution:
Let AB be the ladder of length x.
By Pythagoras theorem,
Answer : x = 17 cm
Q7: Find the area of a rectangle whose length is 144 cm and the length of the
diagonal 145cm.
Answer:
⇒ AD = √(21025 – 20736) = √289
⇒ AD = 17 cm
Thus, area of the rectangle ABCD = 17 × 144 = 2448 cm2.
Conversion Factor is a number used to change one set of units to another, by
multiplyingor dividing. When a conversion is necessary, the appropriate
conversion factor to an equal valuemust be used.
Q8: The distance from the university to home is 10 mi and it usually takes 20 min
to drivethisdistance. Calculate the average speed in meters per second (m/s).
(Note: Average speedisdistance traveled divided by time of travel.) Solution:
1.Calculate average speed. Average speed is distance traveled divided by time
of travel. (Takethis definition as a given for now. Average speed and other motion
concepts are coveredinlater chapters.) In equation form,
Answer:
Note that 100 centimeters divided by 1 meter equals 1 because there are 100
centimeters in a meter. In the calculation, the units you don’t want — meters —
cancel out.
YSON, TRIXIE ANNE
PROBLEM SOLVING
AND DISPLACEMENT
1. The distance between the earth and the sun is on average 93 million
notation.
Given:
93,000,000 mi
12.500,000,000 mi
93,000,000 = 9.3×10⁷ mi
Solution:
= 12.5 × 10⁹
9.3 10⁷
= 1.34×10²
= 134AU
2. Perform the following. Write all answer with correct significant figures. Be
Problem: (8.00×10⁷⁴)(4.11×10¹³)
Solution:
(8.00×10⁷⁴)(9.3×10¹³)
= 8.00 × 10⁷⁴
× 4.11 × 10¹³
= 3.288 × 10⁸⁸
= 4 significant figures
Answer: 0.00543
4. Assume your school is located 2km away from your home. In the morning
you are going to school and in the evening you come back home. In this
entire trip what is the distance travelled and the displacement covered?
Solution:
Distance: 4km
Displacement: 0
5. A car id driven 125.0 km due west, then 65.0 km due south. What is the
F1= 125.0km,W
Fr=?
Formula: c²=a²+b²
Solution:
C²=a²+b²
C²=√a²+b2
Fr²=F1²+F2²
= √(125.0km,W)² + (65.0km,S)²
= √15625km,W² + 4225km, S²
= √19850km,SW
Fr= 141km, SW
6. A plane travels 300 km North and then turns east travels another 400km.
300km,N
400km,E
Formula: c²=a²+b²
Solution:
C²=a²+b²
C²= √a²+b²
= √(300m, N)²+(400m,E)²
= √90000m,N² + 160000m,E²
= √250000²
= 500m,NE
7. Jeramaine run exactly 2 laps around a 400 meter track. Find the distance
and displacement?
Solution:
400 × 2
= 800
Answer:
Distance = 800 m
Displacement = 0
8. Marissa runs north for 37 meters, then turns east and run for another 10
Formula: c²= a² + b²
Solution:
C²= √a² + b²
= √1469m, NE²
= 38.3m, NE
9. The mass of a competition frisbee is 125g. Convert it mass to ounces
Given:
125g
Conversion:
1oz = 28.49g
Solution:
125g × 1oz
28.49g
= 125oz
28.49
=4.41oz
10. A car drives 6mi east, then 12mi north and then 11mi west. What is its
distance travelled?
Given:
6 miles, east
12 miles, north
11 miles, west
Solution:
d = 29 miles